Barron's SAT, 26th edition (2012)

Part 4. TEST YOURSELF

Chapter 10. Five Model SAT Tests

Model Test 3

Answer Sheet—Test 3

Section 1                ESSAY

img

If a section has fewer questions than answer spaces, leave the extra spaces blank.

Section 2

img

Section 3

img

Section 4

img

Section 6

img

Section 7

img

img

Section 8

img

Section 9

img

Section 10

img

Test 3

SECTION 1

Time—25 Minutes

ESSAY

The excerpt appearing below makes a point about a particular topic. Read the passage carefully, and think about the assignment that follows.

Nature (one’s genetic inheritance) affects one’s character and behavior more than nurture (one’s life experiences).

ASSIGNMENT: What are your thoughts on the statement above? Do you agree or disagree with the writer’s assertion that nature or biological inheritance has a greater effect on one’s character and behavior than nurture or life experience does? Compose an essay in which you express your views on this topic. Your essay may support, refute, or qualify the views expressed in the statement. What you write, however, must be relevant to the topic under discussion. Additionally, you must support your viewpoint, indicating your reasoning and providing examples based on your studies and/or experience.

SECTION 2

Time—25 Minutes 24 Questions

Select the best answer to each of the following questions; then blacken the appropriate space on your answer sheet.

Each of the following sentences contains one or two blanks; each blank indicates that a word or set of words has been left out. Below the sentence are five words or phrases, lettered A through E. Select the word or set of words that best completes the sentence.

Example:

Fame is ----; today’s rising star is all too soon tomorrow’s washed-up has-been.

(A) rewarding

(B) gradual

(C) essential

(D) spontaneous

(E) transitory

123

  1. Although in his seventies at the time of the interview, Picasso proved alert and insightful, his faculties ---- despite the inevitable toll of the years.

(A) atrophied

(B) diminished

(C) intact

(D) useless

(E) impaired

  2. While the 1940s are most noted for the development of black modern dance, they are also ---- because they were the last gasp for tap dancing.

(A) irrelevant

(B) unfounded

(C) significant

(D) speculative

(E) contemporary

  3. People who take megadoses of vitamins and minerals should take care: though beneficial in small quantities, in large amounts these substances may have ---- effects.

(A) admirable

(B) redundant

(C) intangible

(D) toxic

(E) minor

  4. The number of black hawks has ---- because the encroachments of humans on their territory have caused them to ---- their customary breeding places.

(A) multiplied..endure

(B) extrapolated..alter

(C) increased..locate

(D) diminished..accept

(E) dwindled..shun

  5. Although Britain’s film makers often produce fine films, they are studiously ---- and rarely aim at a mass market.

(A) commercial

(B) viable

(C) derivative

(D) elitist

(E) collaborative

  6. MacDougall’s former editors remember him as a ---- man whose ---- and exhaustive reporting was worth the trouble.

(A) domineering..wearisome

(B) congenial..pretentious

(C) popular..supercilious

(D) fastidious..garbled

(E) cantankerous..meticulous

  7. The opossum is ---- the venom of snakes in the rattlesnake subfamily and thus views the reptiles not as ---- enemies but as a food source.

(A) vulnerable to..natural

(B) conscious of..mortal

(C) impervious to..lethal

(D) sensitive to..deadly

(E) defenseless against..potential

  8. Breaking with established musical conventions, Stravinsky was ---- composer whose heterodox works infuriated the traditionalists of his day.

(A) a derivative

(B) an iconoclastic

(C) an uncontroversial

(D) a venerated

(E) a trite

Read each of the passages below, and then answer the questions that follow the passage. The correct response may be stated outright or merely suggested in the passage.

Questions 9 and 10 are based on the following passage.

Today, more than ever, Hollywood depends on
adaptations rather than original screenplays for its
story material. This is a far cry from years ago
when studio writers created most of a producer’s

Line (5) scripts. To filmmakers, a best-selling novel has a
peculiar advantage over an original script: already
popular with the public, the story must be a
potential box-office success. Furthermore, it is
usually easier and less time-consuming for a

(10) script writer to adapt a major work than to write
one. The rub for producers is that they pay such
extravagant prices for these properties that the
excess load on the budget often puts the movie
into the red.

  9. In line 6, “peculiar” most nearly means

(A) quaint

(B) bizarre

(C) unfortunate

(D) particular

(E) artistic

10. The primary drawback to basing a screenplay on a best-selling novel is

(A) the amount of time required to create a script based on a novel

(B) the public’s resentment of changes the script writer makes to the novel’s story

(C) the degree of difficulty involved in faithfully adapting a novel for the screen

(D) the desire of studio writers to create their own original scripts

(E) the financial impact of purchasing rights to adapt the novel

Questions 11 and 12 are based on the following passage.

This excerpt from Jack London’s Call of the Wild describes the sled dog Buck’s attempt to rescue his master from the rapids.

When Buck felt Thornton grasp his tail, he
headed for the bank, swimming with all his splendid
strength. From below came the fatal roaring
where the wild current went wilder and was rent

Line (5) in shreds and spray by the rocks that thrust
through like the teeth of an enormous comb. The
suck of the water as it took the beginning of the
last steep pitch was frightful, and Thornton knew
that the shore was impossible. He scraped furiously

(10) over a rock, bruised across a second, and
struck a third with crushing force. He clutched its
slippery top with both hands, releasing Buck, and
above the roar of the churning water shouted:
“Go, Buck! Go!”

11. In line 8, “pitch” most nearly means

(A) high tone

(B) viscous substance

(C) recommendation

(D) intensity

(E) slope

12. The tone of the passage is best described as

(A) lyrical

(B) informative

(C) urgent

(D) ironic

(E) resigned

Questions 13–24 are based on the following passage.

In this adaptation of an excerpt from a short story set in Civil War times, a man is about to be hanged. The first two paragraphs set the scene; the remainder of the passage presents a flashback to an earlier, critical encounter.

A man stood upon a railroad bridge in
Northern Alabama, looking down into the swift
waters twenty feet below. The man’s hands were
behind his back, the wrists bound with a cord. A

Line (5) rope loosely encircled his neck. It was attached to
a stout cross-timber above his head, and the slack
fell to the level of his knees. Some loose boards
laid upon the sleepers supporting the metals of the
railway supplied a footing for him and his executioners—

(10) two private soldiers of the Federal army,
directed by a sergeant, who in civil life may have
been a deputy sheriff. At a short remove upon the
same temporary platform was an officer in the
uniform of his rank, armed. He was a captain. A

(15) sentinel at each end of the bridge stood with his
rifle in the position known as ‘support’—a formal
and unnatural position, enforcing an erect carriage
of the body. It did not appear to be the duty of
these two men to know what was occurring at the

(20) center of the bridge; they merely blockaded the
two ends of the foot plank which traversed it.

The man who was engaged in being hanged
was apparently about thirty-five years of age. He
was a civilian, if one might judge from his dress,

(25) which was that of a planter. His features were
good—a straight nose, firm mouth, broad forehead,
from which his long, dark hair was combed
straight back, falling behind his ears to the collar
of his well-fitting frock coat. He wore a moustache

(30) and pointed beard, but no whiskers; his eyes
were large and dark grey and had a kindly expression
that one would hardly have expected in one
whose neck was in the hemp. Evidently this was
no vulgar assassin. The liberal military code

(35) makes provision for hanging many kinds of people,
and gentlemen are not excluded.

Peyton Farquhar was a well-to-do planter, of
an old and highly respected Alabama family.
Being a slave-owner, and, like other slave-owners,

(40) a politician, he was naturally an original
secessionist and ardently devoted to the Southern
cause. Circumstances had prevented him from
taking service with the gallant army that had
fought the disastrous campaigns ending with the

(45) fall of Corinth, and he chafed under the inglorious
restraint, longing for the release of his energies,
the larger life of the soldier, the opportunity for
distinction. That opportunity, he felt, would come,
as it comes to all in war time. Meanwhile, he did

(50) what he could. No service was too humble for
him to perform in aid of the South, no adventure
too perilous for him to undertake if consistent
with the character of a civilian who was at heart a
soldier, and who in good faith and without too

(55) much qualification assented to at least a part of
the frankly villainous dictum that all is fair in
love and war.

One evening while Farquhar and his wife were
sitting near the entrance to his grounds, a greyclad

(60) soldier rode up to the gate and asked for a
drink of water. Mrs. Farquhar was only too happy
to serve him with her own white hands. While she
was gone to fetch the water, her husband
approached the dusty horseman and inquired

(65) eagerly for news from the front.

“The Yanks are repairing the railroads,” said
the man, “and are getting ready for another
advance. They have reached the Owl Creek
bridge, put it in order, and built a stockade on the

(70) other bank. The commandant has issued an order,
which is posted everywhere, declaring that any
civilian caught interfering with the railroad, its
bridges, tunnels, or trains, will be summarily
hanged. I saw the order.”

(75)      “How far is it to the Owl Creek bridge?”
Farquhar asked.

“About thirty miles.”

“Is there no force on this side of the creek?”

“Only a picket post half a mile out, on the railroad,

(80) and a single sentinel at this end of the
bridge.”

“Suppose a man—a civilian and a student of
hanging—should elude the picket post and perhaps
get the better of the sentinel,” said Farquhar,

(85) smiling, “what could he accomplish?”

The soldier reflected. “I was there a month
ago,” he replied. “I observed that the flood of last
winter had lodged a great quantity of driftwood
against the wooden pier at the end of the bridge.

(90) It is now dry and would burn like tow.”

The lady had now brought the water, which the
soldier drank. He thanked her ceremoniously,
bowed to her husband, and rode away. An hour
later, after nightfall, he repassed the plantation,

(95) going northward in the direction from which he
had come. He was a Yankee scout.

13. In line 11, “civil” most nearly means

(A) polite

(B) individual

(C) legal

(D) collective

(E) nonmilitary

14. In cinematic terms, the first two paragraphs most nearly resemble

(A) a wide-angle shot followed by a close-up

(B) a sequence of cameo appearances

(C) a trailer advertising a feature film

(D) two episodes of an ongoing serial

(E) an animated cartoon

15. In lines 30–33, by commenting on the planter’s amiable physical appearance, the author suggests that

(A) he was innocent of any criminal intent

(B) he seemed an unlikely candidate for execution

(C) the sentinels had no need to fear an attempted escape

(D) the planter tried to assume a harmless demeanor

(E) the eyes are the windows of the soul

16. The author’s tone in discussing “the liberal military code” (line 34) can best be described as

(A) approving

(B) ironic

(C) irked

(D) regretful

(E) reverent

17. Peyton Farquhar would most likely consider which of the following a good example of how a citizen should behave in wartime?

(A) He should use even underhanded methods to support his cause.

(B) He should enlist in the army without delay.

(C) He should turn to politics as a means of enforcing his will.

(D) He should avoid involving himself in disastrous campaigns.

(E) He should concentrate on his duties as a planter.

18. In line 52, “consistent” most nearly means

(A) unfailing

(B) agreeable

(C) dependable

(D) constant

(E) compatible

19. In line 55, “qualification” most nearly means

(A) competence

(B) eligibility

(C) restriction

(D) reason

(E) liability

20. It can be inferred from lines 61 and 62 that Mrs. Farquhar is

(A) sympathetic to the Confederate cause

(B) uninterested in news of the war

(C) too proud to perform menial tasks

(D) reluctant to ask her slaves to fetch water

(E) inhospitable by nature

21. Farquhar’s inquiry about what a man could accomplish (lines 82–85) illustrates which aspect of his character?

(A) Morbid longing for death

(B) Weighty sense of personal responsibility

(C) Apprehension about his family’s future

(D) Keenly inquisitive intellect

(E) Romantic vision of himself as a hero

22. From Farquhar’s exchange with the soldier (lines 75–90), we can infer that Farquhar most likely is going to

(A) sneak across the bridge to join the Confederate forces

(B) attempt to burn down the bridge to halt the Yankee advance

(C) remove the driftwood blocking the Confederates’ access to the bridge

(D) attack the stockade that overlooks the Owl Creek bridge

(E) undermine the pillars that support the railroad bridge

23. As used in the next-to-last paragraph, “tow” is

(A) an act of hauling something

(B) a tugboat

(C) a railroad bridge

(D) a highly combustible substance

(E) a picket post

24. We may infer from lines 93–96 that

(A) the soldier has deserted from the Southern army

(B) the soldier has lost his sense of direction

(C) the scout has been tempting Farquhar into an unwise action

(D) Farquhar knew the soldier was a Yankee scout

(E) the soldier returned to the plantation unwillingly

S T O P

YOU MAY GO BACK AND REVIEW THIS SECTION IN THE REMAINING TIME, BUT DO NOT WORK IN ANY OTHER SECTION UNTIL TOLD TO DO SO.

SECTION 3

Time—25 Minutes 20 Questions

For each problem in this section determine which of the five choices is correct and blacken the corresponding choice on your answer sheet. You may use any blank space on the page for your work.

Notes:

• You may use a calculator whenever you think it will be helpful.

• Only real numbers are used. No question or answer on this test involves a complex or imaginary number.

• Use the diagrams provided to help you solve the problems. Unless you see the words “Note: Figure not drawn to scale” under a diagram, it has been drawn as accurately as possible. Unless it is stated that a figure is three-dimensional, you may assume it lies in a plane.

• For any function f, the domain, unless specifically restricted, is the set of all real numbers for which f(x) is also a real number.

Reference Information

123

123

  1. If 2x + 4x + 6x = –12, then x =

(A) –1

(B) – 123

(C)    0

(D)    123

(E)    1

  2. What is the product of 1.1 and 1.9 rounded to the nearest tenth?

(A) 1.5

(B) 1.7

(C) 2.0

(D) 2.1

(E) 3.0

123

  3. In the figure above, lines k and 123 are parallel, and line k passes through D, one of the corners of square ABCD. What is the value of w?

(A) 30

(B) 40

(C) 45

(D) 50

(E) 60

Questions 4 and 5 refer to the following table.

The data show the number of students in each of the five fifth-grade classes at Taft Elementary School and, for each class, the number of students in the school band.

123

  4. What is the average (arithmetic mean) number of students per class?

(A) 23

(B) 24

(C) 24.5

(D) 25

(E) 26

  5. Which class has the highest percent of students in the band?

(A) A

(B) B

(C) C

(D) D

(E) E

  6. Steve took a bike trip in which he covered half the total distance on Monday. After going 100 kilometers on Tuesday, he determined that he still had 10% of the trip to complete. What was the total length, in kilometers, of the trip?

(A) 200

(B) 250

(C) 400

(D) 500

(E) 600

  7. A number, x, is chosen at random from the set of positive integers less than 10. What is the probability that 123

(A) 123

(B) 123

(C) 123

(D) 123

(E) 123

  8. The members of the French Club conducted a fund-raising drive. The average (arithmetic mean) amount of money raised per member was $85. Then Jean joined the club and raised $50. This lowered the average to $80. How many members were there before Jean joined?

(A) 4

(B) 5

(C) 6

(D) 7

(E) 8

  9. RS, and T are points with RT = 2RS. Which of the following could be true?

  I. RS, and T are the vertices of a right triangle.

 II. RS, and T are three of the vertices of a square.

III. RS, and T all lie on the circumference of a circle.

(A) I only

(B) III only

(C) I and II only

(D) I and III only

(E) I, II, and III

10. There are 12 men on a basketball team, and in a game 5 of them play at any one time. If the game is 1 hour long, and if each man plays exactly the same amount of time, how many minutes does each man play?

(A) 10

(B) 12

(C) 24

(D) 25

(E) 30

11. The volume of pitcher I is A ounces, and the volume of pitcher II is B ounces, with B > A. If pitcher II is full of water and pitcher I is empty, and if just enough water is poured from pitcher II to fill pitcher I, what fraction of pitcher II is now full?

(A) 123

(B) 123

(C) 123

(D) 123

(E) 123

123

12. In the figure above, w + x + y + z =

(A) 140

(B) 280

(C) 300

(D) 320

(E) 360

13. What is the greatest value of x that is a solution of the following equation?

|x – 5| + 10 = 15

(A)   0

(B)   5

(C) 10

(D) 20

(E) 30

x + y = 10    y + z = 15    x + z = 17

14. What is the average (arithmetic mean) of xy, and z?

(A)   7

(B) 14

(C) 15

(D) 21

(E) It cannot be determined from the information given.

15. A number of people boarded a bus at the terminal. At the first stop, half of the passengers got off and 1 got on. At the second stop, 123 of the passengers on the bus got off and 1 got on. If the bus then had 15 passengers, how many were there when the bus left the terminal?

(A) 40

(B) 48

(C) 58

(D) 60

(E) 62

16. Thirty years ago, Mr. and Mrs. Lopez purchased a house. On average, the value of the house has doubled every 6 years. If the house is worth $320,000 today, what did they pay for it 30 years ago?

(A) $     5,000

(B) $   10,000

(C) $   64,000

(D) $160,000

(E) $320,000 × 25

123

Note: Figure not drawn to scale.

17. Which of the following statements concerning the triangle in the figure above must be true?

  I. c = 80 – a

 II. c = b – 50

III. a + b = c + d

(A) I only

(B) II only

(C) I and II only

(D) I and III only

(E) I, II, and III

18. If c carpenters can build a garage in d days, how many days will it take e carpenters, working at the same rate, to build 2 garages?

(A) 123

(B) 123

(C) 123

(D) 123

(E) 123

19. If 123

(A) –2

(B) –0

(C) 2

(D) 123

(E) 123

20. Let A = total area of five circles of radius r, and let B = total area of three circles of radius s. If A = B, then 123

(A) 123

(B) 123

(C) 123

(D) 123

(E) 123

S T O P

YOU MAY GO BACK AND REVIEW THIS SECTION IN THE REMAINING TIME, BUT DO NOT WORK IN ANY OTHER SECTION UNTIL TOLD TO DO SO.

SECTION 4

Time—25 Minutes 35 Questions

Select the best answer to each of the following questions; then blacken the appropriate space on your answer sheet.

Some or all parts of the following sentences are underlined. The first answer choice, (A), simply repeats the underlined part of the sentence. The other four choices present four alternative ways to phrase the underlined part. Select the answer that produces the most effective sentence, one that is clear and exact, and blacken the appropriate space on your answer sheet. In selecting your choice, be sure that it is standard written English, and that it expresses the meaning of the original sentence.

Example:

The first biography of author Eudora Welty came out in 1998 and she was 89 years old at the time.

(A) and she was 89 years old at the time

(B) at the time when she was 89

(C) upon becoming an 89 year old

(D) when she was 89

(E) at the age of 89 years old

123

  1. Unable to see more than three inches in front of her nose without corrective lenses, Mary’s search for her missing glasses was frantic.

(A) Mary’s search for her missing glasses was frantic

(B) Mary’s frantic search was for her missing glasses

(C) Mary frantically searched for her missing glasses

(D) her missing glasses were what Mary frantically searched for

(E) her missing glasses was that for which Mary frantically searched

  2. Ron liked to play word games, of which he found crossword puzzles particularly satisfying.

(A) games, of which he found crossword puzzles particularly satisfying

(B) games, and it was crossword puzzles that particularly found satisfaction

(C) games, particularly satisfying to him were crossword puzzles

(D) games; he found crossword puzzles particularly satisfying

(E) games; the satisfaction of crossword puzzles particularly

  3. Martin Luther King Jr.’s influence had a strong impact on the members of the Southern Christian Leadership Conference, especially Jesse Jackson and Ralph Abernathy.

(A) Martin Luther King Jr.’s influence had a strong impact on the members

(B) Martin Luther King Jr.’s influence had a strong impact regarding the members

(C) Martin Luther King Jr. strongly influenced members

(D) The influence of Martin Luther King Jr. was strong on the members

(E) Martin Luther King Jr.’s influence strongly impacted members

  4. Raise High the Roofbeam, Carpenters is a novel written by the notoriously reclusive J. D. Salinger.

(A) Raise High the Roofbeam, Carpenters is a novel written by the notoriously reclusive J. D. Salinger.

(B) Raise High the Roofbeam, Carpenters were a novel written by the notorious reclusive J. D. Salinger.

(C) Raise High the Roofbeam, Carpenters were a novel that the notoriously reclusive J. D. Salinger wrote.

(D) As a notorious recluse, J. D. Salinger has written a novel that is called Raise High the Roofbeam, Carpenters.

(E) Raise High the Roofbeam, Carpenters is the name of a novel that was written by the notorious reclusive J. D. Salinger.

  5. Fans of Donald Trump’s reality television show The Apprentice have described it as simultaneously infuriating because of Trump’s arrogance but Trump’s shrewdness still has a fascination.

(A) but Trump’s shrewdness still has a fascination

(B) and Trump’s shrewdness still is fascinating

(C) and Trump is fascinatingly shrewd

(D) and fascinating because of Trump’s shrewdness

(E) while Trump is so shrewd that he fascinates them

  6. That it is deemed necessary to shield television viewers from ads concerning pressing public issues while they are being bombarded with commercial pitches for beer and sports utility vehicles is a sad commentary on the state of our culture and of our democracy.

(A) they are being bombarded with commercial pitches for beer and sports utility vehicles is a sad commentary on the state of our culture and of our democracy

(B) they had been bombarded with commercial pitches for beer and sports utility vehicles sadly is a commentary on the state of our culture and of our democracy

(C) it is bombarded with commercial pitches for beer and sports utility vehicles is a sad commentary on the state of our culture and of our democracy

(D) they are being bombarded with commercial pitches for beer and sports utility vehicles are sad commentaries on the state of our culture and of our democracy

(E) they are bombarding with commercial pitches for beer and sports utility vehicles is a sad commentary on the state of our culture and of our democracy

  7. There is simply no way one can avoid conflict; hence, if you must fight, fight to win.

(A) There is simply no way one can avoid conflict; hence,

(B) In no way can one simply avoid conflict; hence,

(C) You cannot avoid conflict; hence,

(D) There is simply no way one can avoid conflict; however,

(E) There is simply no way in which you may avoid conflict; consequently,

  8. The federal Fish and Wildlife Service is expected to rule this week on whether to protect beluga sturgeon under the Endangered Species Act.

(A) is expected to rule this week on whether to protect beluga sturgeon under the Endangered Species Act

(B) are expected to rule this week on whether to protect beluga sturgeon under the Endangered Species Act

(C) have been expected to rule this week on whether to protect beluga sturgeon under the Endangered Species Act

(D) is expected to rule this week about the protecting of beluga sturgeon by means of the Endangered Species Act

(E) is being expected to rule this week on whether or not they should protect beluga sturgeon under the Endangered Species Act

  9. In most states where local property taxes fund the public schools, communities with strong tax bases from commercial property can support its schools while maintaining low property tax rates.

(A) with strong tax bases from commercial property can support its schools while maintaining low property tax rates

(B) that have strong tax bases from commercial property can support their schools and maintaining low property tax rates

(C) with strong tax bases from commercial property could have supported its schools while maintaining low property tax rates

(D) with strong tax bases from commercial property can support their schools while maintaining low property tax rates

(E) with strong tax bases from commercial property could of supported its schools and the maintenance of low property tax rates

10. The drop in interest rates, especially for home mortgages, have encouraged prospective buyers and applied for loans.

(A) have encouraged prospective buyers and applied for loans

(B) have encouraged prospective buyers and loans have been applied for

(C) have encouraged prospective buyers; therefore, they applied for loans

(D) has encouraged prospective buyers, that they applied for loans

(E) has encouraged prospective buyers to apply for loans

11. The bridge between San Francisco and Marin County, California, is actually painted a reddish orange, while being called the Golden Gate.

(A) is actually painted a reddish orange, while being

(B) although actually painted a reddish orange, is

(C) whose paint is actually a reddish orange, while it is

(D) being actually painted a reddish orange caused it to be

(E) which is actually painted a reddish orange, while being

The sentences in this section may contain errors in grammar, usage, choice of words, or idioms. Either there is just one error in a sentence or the sentence is correct. Some words or phrases are underlined and lettered; everything else in the sentence is correct.

If an underlined word or phrase is incorrect, choose that letter; if the sentence is correct, select No error. Then blacken the appropriate space on your answer sheet.

Example:

The region has a climate 123 that plants 123 rarely 123 more than twelve inches 123

123

12. 123 many people are 123 the web site, 123 well known to shoppers 123 comparing prices before they make purchases online. 123

13. The museum’s juxtaposition images Weegee’s images with images his competitors images the sophistication of his images photography. 123

14. 123 the world’s largest rodent is the capybara, a 123 mammal found throughout 123 South America. 123

15. 123 rain has long been used as a water source in areas 123 well water is unavailable 123 tainted, the amounts collected are usually small and rarely 123 consumption without treatment. 123

16. Though 123 popular histories of World War II, black soldiers fought 123 whites in the war’s final year 123 the American Revolution. 123

17. Joan of Arc had a 123 save France, a 123 performing miracles, and 123 endure great suffering 123 her faith. 123

18. 123 the thorough investigation after the assassination, 123 little is 123 the motivations of the killer 123 down the prime minister. 123

19. Cream, like 123 dairy products that spoil 123 123 to be kept 123 refrigeration. 123

20. 123 California’s Tevis Cup race and Australia’s Quilty Cup are very similar equestrian events, 123 the Tevis Cup 123 challenge to both horses and riders. 123

21. During the 1920s, members of the white literary establishment 123 interest in the movement of black writers 123 came to be 123 the Harlem Renaissance. 123

22. 123 Whitman’s verses, 123 are wholly 123 absence of rhyme. 123

23. Her interest in fine food 123 to visit ethnic foodmarkets throughout the region 123 as 123 at the nearby Culinary Institute.123

24. The 123 advantages this proposed merger can 123 our firm 123 any of the potential disadvantages 123 opponents of the consolidation. 123

25. 123 the candidate made 123 use of the Internet to raise funds for his campaign; 123 he went on to 123 conventional fund-raising methods. 123

26. A images downpour that images the poolside area where the sunbathers images caused images to scatter. 123

27. It is likely that the Coen brothers’ latest movie, 123 scheduled 123 in time for Thanksgiving, 123 postponed until summer because of 123 postproduction difficulties.123

28. 123 his lifetime, Degas exhibited 123 one piece of sculpture, Little Dancer, Aged Fourteen123 123 in 1881 in the sixth exhibition of Impressionist art in Paris. 123

29. The differences between Locke’s world view and 123 less from a dispute about the function of government 123 a dispute about the 123 mankind. 123

The passage below is the unedited draft of a student’s essay. Parts of the essay need to be rewritten to make the meaning clearer and more precise. Read the essay carefully.

The essay is followed by six questions about changes that might improve all or part of the organization, development, sentence structure, use of language, appropriateness to the audience, or use of standard written English. In each case, choose the answer that most clearly and effectively expresses the student’s intended meaning. Indicate your choice by blackening the corresponding space on the answer sheet.

[1] Although some people believe that certain celebrations have no point, celebrations are one of the few things that all people have in common. [2] They take place everywhere. [3] Listing all of them would be an impossible task. [4] People of all kinds look forward to celebrations for keeping traditions alive for generation after generation. [5] Those who criticize celebrations do not understand the human need to preserve tradition and culture.

[6] In the Muslim religion, the Ead is a celebration.

[7] It begins as soon as Ramadan (the fasting month) is over. [8] During the Ead, families gather together. [9] New clothes are bought for children, and they receive money from both family and friends. [10] Also, each family, if they can afford it, slaughters a sheep or a cow.

[11] They keep a small fraction of the meat, and the rest must give to the poor. [12] They also donate money to a mosque.

[14] Many celebrations involve eating meals. [15] In the United States, people gather together on Thanksgiving to say thank you for their blessings by having a huge feast with turkey, sweet potatoes, and cranberry sauce. [16] Christmas and Easter holiday dinners are a custom in the Christian religion. [17] They have a roast at Christmas. [18] At Easter they serve ham. [19] The Jewish people celebrate Passover with a big meal called a seder. [20] They say prayers, drink wine, and sing songs to remember how Jews suffered centuries ago when they escaped from slavery in Egypt.

[21] A celebration is held each year to honor great people like Dr. Martin Luther King. [22] His birthday is celebrated because of this man’s noble belief in equality of all races. [23] People wish to remember not only his famous speeches, including “I Have A Dream,” but also about him being assassinated in Memphis in 1968. [24] He died while fighting for the equality of minorities. [25] Unlike religious celebrations, celebrations for great heroes like Martin Luther King are for all people everywhere in the world. [26] He is a world-class hero and he deserved the Nobel Prize for Peace that he won.

30. To improve the unity of the first paragraph, which of the following is the best sentence to delete?

(A) Sentence 1

(B) Sentence 2

(C) Sentence 3

(D) Sentence 4

(E) Sentence 5

31. Which is the best revision of sentence 9 below?
New clothes are bought for children, and they receive money from both family and friends.

(A) New clothes are bought for children, and they receive money from both family and friends.

(B) The children receive new clothes and gifts of money from family and friends.

(C) Receiving new clothes, money is also given by family and friends.

(D) Gifts are given to the children of new clothes and money by family and friends.

(E) Parents buy new clothes for their children, and family and friends also give money to them.

32. In the context of the third paragraph, which is the best way to combine sentences 16, 17, and 18?

(A) A roast at Christmas, ham at Easter—that’s what Christians eat.

(B) Christians customarily serve a roast for Christmas dinner, at Easter ham is eaten.

(C) At customary holiday dinners, Christians eat a roast at Christmas and ham is for Easter dinner.

(D) Christians often celebrate the Christmas holiday with a roast for dinner and Easter with a traditional ham.

(E) Christmas and Easter dinners are the custom in the Christian religion, where they have a roast at Christmas and ham at Easter.

33. In an effort to provide a more effective transition between paragraphs 3 and 4, which of the following would be the best revision of sentence 21 below?

A celebration is held each year to honor great people like Dr. Martin Luther King.

(A) There are also some celebrations to honor great people like Dr. Martin Luther King.

(B) Martin Luther King is also celebrated in the United States.

(C) In the United States, celebrating to honor great people like Dr. Martin Luther King has become a tradition.

(D) In addition to observing religious holidays, people hold celebrations to honor great leaders like Dr. Martin Luther King.

(E) Besides holding religion-type celebrations, celebrations to honor great people like Dr. Martin Luther King are also held.

34. Which is the best revision of the underlined segment of sentence 23 below?

People wish to remember not only his famous speeches, including “I Have A Dream,” but also about him being assassinated in Memphis in 1968.

(A) that his assassination occurred

(B) about his being assassinated

(C) the fact that he was assassinated

(D) about the assassination, too,

(E) his assassination

35. In the context of the essay as a whole, which one of the following best explains the main function of the last paragraph?

(A) To summarize the main idea of the essay

(B) To refute a previous argument stated in the essay

(C) To give an example

(D) To provide a solution to a problem

(E) To evaluate the validity of the essay’s main idea

S T O P

YOU MAY GO BACK AND REVIEW THIS SECTION IN THE REMAINING TIME, BUT DO NOT WORK IN ANY OTHER SECTION UNTIL TOLD TO DO SO.

SECTION 6

Time—25 Minutes 24 Questions

Select the best answer to each of the following questions; then blacken the appropriate space on your answer sheet.

Each of the following sentences contains one or two blanks; each blank indicates that a word or set of words has been left out. Below the sentence are five words or phrases, lettered A through E. Select the word or set of words that best completes the sentence.

Example:

Fame is ----; today’s rising star is all too soon tomorrow’s washed-up has-been.

(A) rewarding

(B) gradual

(C) essential

(D) spontaneous

(E) transitory

123

  1. The critics were distressed that an essayist of such glowing ---- could descend to writing such dull, uninteresting prose.

(A) obscurity

(B) ill-repute

(C) shallowness

(D) promise

(E) amiability

  2. Although Henry was not in general a sentimental man, occasionally he would feel a touch of ---- for the old days and would contemplate making a brief excursion to Boston to revisit his childhood friends.

(A) exasperation

(B) chagrin

(C) nostalgia

(D) lethargy

(E) anxiety

  3. We had not realized how much people ---- the library’s old borrowing policy until we received complaints once it had been --.

(A) enjoyed..continued

(B) disliked..administered

(C) respected..imitated

(D) ignored..lauded

(E) appreciated..superseded

  4. Even though the basic organization of the brain does not change after birth, details of its structure and function remain ---- for some time, particularly in the cerebral cortex.

(A) plastic

(B) immutable

(C) essential

(D) unavoidable

(E) static

  5. Lavish in visual beauty, the film Lawrence of Arabia also boasts ---- of style: it knows how much can be shown in a shot, how much can be said in a few words.

(A) extravagance

(B) economy

(C) autonomy

(D) frivolity

(E) arrogance

Read the passages below, and then answer the questions that follow them. The correct response may be stated outright or merely suggested in the passages.

Questions 6–9 are based on the following passages.

Passage 1

In 1979, when the World Health Organization
declared that smallpox had finally been eradicated,
few, if any, people recollected the efforts
of an eighteenth-century English aristocrat to

Line (5) combat the then-fatal disease. As a young woman,
Lady Mary Wortley Montagu had suffered
severely from smallpox. In Turkey, she observed
the Eastern custom of inoculating people with a
mild form of the pox, thereby immunizing them, a

(10) practice she later championed in England. The
Turks, she wrote home, even held house parties
during which inoculated youngsters played
together happily until they came down with the
pox, after which they convalesced together.

Passage 2

(15)       Who was Onesimus? New Testament students
say that Onesimus was a slave converted to
Christianity by the apostle Paul. In doing so, they
ignore the claims of another slave named
Onesimus, an African, who in 1721 helped stem a

(20) smallpox epidemic threatening the city of Boston.
Asked by his owner, Cotton Mather, whether he
had ever had smallpox, Onesimus responded,
“Yes, and no,” for as a child he had been
intentionally infected with smallpox in a process called

(25) inoculation and had become immune to the
disease. Emboldened by Onesimus’s account,
Mather led a successful campaign to inoculate
Bostonians against the dread disease.

  6. The primary purpose of both passages is to

(A) celebrate the total eradication of smallpox

(B) challenge the achievements of Lady Mary Wortley Montagu

(C) remind us that we can learn from foreign cultures

(D) show that smallpox was a serious problem in the eighteenth century

(E) call attention to neglected historical figures

  7. According to Passage 1, Lady Mary’s efforts to combat smallpox in England came about

(A) as a direct result of her childhood exposure to the disease

(B) as part of a World Health Organization campaign against the epidemic

(C) in response to the migration of Turks to England

(D) as a consequence of her travels in the East

(E) in the face of opposition from the medical profession

  8. In Passage 1, the author uses the word “even” (line 11) primarily to

(A) exaggerate the duration of the house parties

(B) emphasize the widespread acceptance of the inoculation procedure

(C) indicate the most appropriate setting for treatment

(D) encourage her readers to travel to Turkey

(E) underscore the dangers of English methods for treating the disease

  9. Lady Mary Wortley Montagu (lines 7–14, Passage 1) and Cotton Mather (lines 21–28, Passage 2) serve as examples of

(A) scientists who were authorities on epidemiology

(B) individuals who advocated a foreign medical practice

(C) travelers who brought back word of new therapeutic techniques

(D) slave owners who had the wisdom to learn from their slaves

(E) writers whose works reveal an ignorance of current medical traditions

Questions 10–15 are based on the following passage.

Are Americans today overworked? The following passage is excerpted from a book published in 1991 on the unexpected decline of leisure in American life.

Faith in progress is deep within our culture.
We have been taught to believe that our lives are
better than the lives of those who came before us.
The ideology of modern economics suggests that

Line (5) material progress has yielded enhanced satisfaction
and well-being. But much of our confidence
about our own well being comes from the
assumption that our lives are easier than those of
earlier generations. I have already disputed the

(10) notion that we work less than medieval European
peasants, however poor they may have been. The
field research of anthropologists gives another
view of the conventional wisdom.

The lives of so-called primitive peoples are

(15) commonly thought to be harsh—their existence
dominated by the “incessant quest for food.” In
fact, primitives do little work. By contemporary
standards, we’d have to judge them very lazy. If
the Kapauku of Papua work one day, they do no

(20) labor on the next. !Kung Bushmen put in only
two and a half days per week and six hours per
day. In the Sandwich Islands of Hawaii, men
work only four hours per day. And Australian
aborigines have similar schedules. The key to

(25) understanding why these “stone age peoples” fail
to act like us—increasing their work effort to get
more things—is that they have limited desires. In
the race between wanting and having, they have
kept their wanting low—and, in this way, ensure

(30) their own kind of satisfaction. They are materially
poor by contemporary standards, but in at least
one dimension—time—we have to count them
richer.

I do not raise these issues to imply that we

(35) would be better off as Polynesian natives or
medieval peasants. Nor am I arguing that
“progress” has made us worse off. I am, instead,
making a much simpler point. We have paid a
price for prosperity. Capitalism has brought a

(40) dramatically increased standard of living, but at the
cost of a much more demanding worklife. We are
eating more, but we are burning up those calories
at work. We have color televisions and compact
disc players, but we need them to unwind after a

(45) stressful day at the office. We take vacations, but
we work so hard throughout the year that they
become indispensable to our sanity. The conventional
wisdom that economic progress has given
us more things as well as more leisure is difficult

(50) to sustain.

10. According to the author, we base our belief that American people today are well off on the assumption that

(A) America has always been the land of opportunity

(B) Americans particularly deserve to be prosperous

(C) people elsewhere have an inferior standard of living

(D) people elsewhere envy the American way of life

(E) our faith in progress will protect us as a nation

11. The author regards “the conventional wisdom” (line 13) with

(A) resentment

(B) skepticism

(C) complacency

(D) apprehension

(E) bewilderment

12. In lines 18–22, the Kapauku tribesmen and the !Kung Bushmen are presented as examples of

(A) malingerers who turn down opportunities to work

(B) noble savages with little sense of time

(C) people who implicitly believe in progress

(D) people unmotivated by a desire for consumer goods

(E) people obsessed by their constant search for food

13. In line 34, “raise” most nearly means

(A) elevate

(B) increase

(C) nurture

(D) bring up

(E) set upright

14. The primary purpose of the passage is to

(A) dispute an assumption

(B) highlight a problem

(C) ridicule a theory

(D) answer a criticism

(E) counter propaganda

15. The last four sentences of the passage (lines 41–50) provide

(A) a recapitulation of a previously made argument

(B) an example of the argument that has been proposed earlier

(C) a series of assertions and qualifications with a conclusion

(D) a reconciliation of two opposing viewpoints

(E) a reversal of the author’s original position

Questions 16–24 are based on the following passage.

The following passage, written in the twentieth century, is taken from a discussion of John Webster’s seventeenth-century drama “The Duchess of Malfi.”

The curtain rises; the Cardinal and Daniel de
Bosola enter from the right. In appearance, the
Cardinal is something between an El Greco cardinal
and a Van Dyke noble lord. He has the tall,

Line (5) spare form—the elongated hands and features—of
the former; the trim pointed beard, the imperial
repose, the commanding authority of the latter.
But the El Greco features are not really those of
asceticism or inner mystic spirituality. They are

(10) the index to a cold, refined but ruthless cruelty in
a highly civilized controlled form. Neither is the
imperial repose an aloof mood of proud detachment.
It is a refined expression of satanic pride of
place and talent.

(15) To a degree, the Cardinal’s coldness is
artificially cultivated. He has defined himself against
his younger brother Duke Ferdinand and is the
opposite to the overwrought emotionality of the
latter. But the Cardinal’s aloof mood is not one of

(20) bland detachment. It is the deliberate detachment
of a methodical man who collects his thoughts
and emotions into the most compact and formidable
shape—that when he strikes, he may strike
with the more efficient and devastating force. His

(25) easy movements are those of the slowly circling
eagle just before the swift descent with the
exposed talons. Above all else, he is a man who
never for a moment doubts his destined authority
as a governor. He derisively and sharply rebukes

(30) his brother the Duke as easily and readily as he
mocks his mistress Julia. If he has betrayed his
hireling Bosola, he uses his brother as the tool to
win back his “familiar.” His court dress is a long
brilliant scarlet cardinal’s gown with white cuffs

(35) and a white collar turned back over the red, both
collar and cuffs being elaborately scalloped and
embroidered. He wears a small cape, reaching
only to the elbows. His cassock is buttoned to the
ground, giving a heightened effect to his already

(40) tall presence. Richelieu would have adored his
neatly trimmed beard. A richly jeweled and ornamented
cross lies on his breast, suspended from
his neck by a gold chain.

Bosola, for his part, is the Renaissance “familiar”

(45) dressed conventionally in somber black with
a white collar. He wears a chain about his neck, a
suspended ornament, and a sword. Although a
“bravo,” he must not be thought of as a leather-jacketed,
heavy-booted tough, squat and swarthy.

(50) Still less is he a sneering, leering, melodramatic
villain of the Victorian gaslight tradition. Like his
black-and-white clothes, he is a colorful contradiction,
a scholar-assassin, a humanist-hangman;
introverted and introspective, yet ruthless in

(55) action; moody and reluctant, yet violent. He is a
man of scholarly taste and subtle intellectual
discrimination doing the work of a hired ruffian. In
general effect, his impersonator must achieve
suppleness and subtlety of nature, a highly complex,

(60) compressed, yet well restrained intensity of
temperament. Like Duke Ferdinand, he is inwardly
tormented, but not by undiluted passion. His dominant
emotion is an intellectualized one: that of
disgust at a world filled with knavery and folly,

(65) but in which he must play a part and that a lowly,
despicable one. He is the kind of rarity that
Browning loved to depict in his Renaissance
monologues.

16. The primary purpose of the passage appears to be to

(A) provide historical background on the Renaissance church

(B) describe ecclesiastical costuming and pageantry

(C) analyze the appearances and moral natures of two dramatic figures

(D) explain why modern audiences enjoy The Duchess of Malfi

(E) compare two interpretations of a challenging role

17. In line 5, “spare” most nearly means

(A) excessive

(B) superfluous

(C) pardonable

(D) lean

(E) inadequate

18. In lines 24–27, the author most likely compares the movements of the Cardinal to those of a circling eagle in order to emphasize his

(A) flightiness

(B) love of freedom

(C) eminence

(D) spirituality

(E) mercilessness

19. The Cardinal’s “satanic pride of place” (lines 13 and 14) refers to his glorying in his

(A) faith

(B) rank

(C) residence

(D) immobility

(E) wickedness

20. In line 48, “bravo” most nearly means

(A) a courageous man

(B) a national hero

(C) a clergyman

(D) a humanist

(E) a mercenary killer

21. In describing Bosola (lines 44–68), the author chiefly uses which of the following literary techniques?

(A) Rhetorical questions

(B) Unqualified assertions

(C) Comparison and contrast

(D) Dramatic irony

(E) Literary allusion

22. In lines 56–57, “discrimination” most nearly means

(A) prejudice

(B) villainy

(C) discretion

(D) favoritism

(E) discernment

23. According to lines 61–66, why does Bosola suffer torments?

(A) His master, the Cardinal, berates him for performing his duties inadequately.

(B) He feels intense compassion for the pains endured by the Cardinal’s victims.

(C) He is frustrated by his inability to attain a higher rank in the church.

(D) He feels superior to the villainy around him, yet must act the villain himself.

(E) He lacks the intellectual powers for scholarly success, but cannot endure common fools.

24. The author of the passage assumes that the reader is

(A) familiar with the paintings of El Greco and Van Dyke

(B) disgusted with a world filled with cruelty and folly

(C) ignorant of the history of the Roman Catholic Church

(D) uninterested in psychological distinctions

(E) unacquainted with the writing of Browning

S T O P

YOU MAY GO BACK AND REVIEW THIS SECTION IN THE REMAINING TIME, BUT DO NOT WORK IN ANY OTHER SECTION UNTIL TOLD TO DO SO.

SECTION 7

Time—25 Minutes 18 Questions

You have 25 minutes to answer the 8 multiple-choice questions and 10 student-produced response questions in this section. For each multiple-choice question, determine which of the five choices is correct and blacken the corresponding choice on your answer sheet. You may use any blank space on the page for your work.

Notes:

• You may use a calculator whenever you think it will be helpful.

• Only real numbers are used. No question or answer on this test involves a complex or imaginary number.

• Use the diagrams provided to help you solve the problems. Unless you see the words “Note: Figure not drawn to scale” under a diagram, it has been drawn as accurately as possible. Unless it is stated that a figure is three-dimensional, you may assume it lies in a plane.

• For any function f, the domain, unless specifically restricted, is the set of all real numbers for which f (x) is also a real number.

Reference Information

123

123

  1. If (w + 12) – 12 = 12, w =

(A) –12

(B)     0

(C)   12

(D)   24

(E)   36

  2. If 24 of the 40 students in a club are girls, what is the ratio of boys to girls in the club?

(A) 2:5

(B) 3:5

(C) 2:3

(D) 3:2

(E) 5:2

3. If 25 – 3 123= 7, what is the value of x?

(A) –36

(B)   –6

(C)     6

(D)   36

(E) There is no value of x that satisfies the equation

  4. What is the area of the circle whose center is at O (2, –2) and that passes through point P (3, 3)?

(A)   2123

(B)   6123

(C) 10123

(D) 25123

(E) 26123

  5. At Music Outlet the regular price for a CD is d dollars. How many CDs can be purchased for m dollars when the CDs are on sale at 50% off the regular price?

(A) 123

(B) 123

(C) 123

(D) 123

(E) 123

  6. If f (x) = 9x + 9x, what is the value of 123

(A) 3

(B) 6

(C) 7.5

(D) 9

(E) 9.9

7. Which of the following is equivalent to 123

(A) 2

(B) 123

(C) 123

(D) 123

(E) 123

  8. The first term of sequence I is 2, and each subsequent term is 2 more than the preceding term. The first term of sequence II is 2 and each subsequent term is 2 times the preceding term. What is the ratio of the 32nd term of sequence II to the 32nd term of sequence I?

(A) 1

(B) 2

(C) 226

(D) 227

(E) 232

Directions for Student-Produced Response Questions (Grid-ins)

In questions 9–18, first solve the problem, and then enter your answer on the grid provided on the answer sheet. The instructions for entering your answers are as follows:

• First, write your answer in the boxes at the top of the grid.

• Second, grid your answer in the columns below the boxes.

• Use the fraction bar in the first row or the decimal point in the second row to enter fractions and decimal answers.

123

• Grid only one space in each column.

• Entering the answer in the boxes is recommended as an aid in gridding, but is not required.

• The machine scoring your exam can read only what you grid, so you must grid in your answers correctly to get credit.

• If a question has more than one correct answer, grid in only one of these answers.

• The grid does not have a minus sign, so no answer can be negative.

• A mixed number must be converted to an improper fraction or a decimal before it is gridded. Enter 1123 as 5/4 or 1.25; the machine will interpret 1 1/4 as 123 and mark it wrong.

• All decimals must be entered as accurately as possible. Here are the three acceptable ways of gridding

123

• Note that rounding to .273 is acceptable, because you are using the full grid, but you would receive no credit for .3 or .27, because these answers are less accurate.

123

  9. In the figure above, what is the area of 123ABC?

10. For any numbers, a and ba images b is defined to be (a2 + b2) − (a2 − b2). What is the value of 6 images 7?

123

11. A square, not shown, has the same perimeter as the quadrilateral above. What is the length of a side of the square?

12. A factory can produce 1 gizmo every 333 seconds. How many hours will it take to produce 40 gizmos?

13. If the average (arithmetic mean) of ab, and 10 is 100, what is the average of a and b?

14. If the rent on an apartment goes up 10% every year, next year’s rent will be how many times last year’s rent?

15. How many 5-digit positive integers are there that satisfy the following conditions?
• Each digit is a 1, 2, 3, 4, or 5.
• No digit is repeated.
• The first and last digits are both odd.

16. When 25 students took a quiz, the grades they earned ranged from 2 to 10. If exactly 22 of them passed, by earning a grade of 7 or higher, what is the highest possible average (arithmetic mean) the class could have earned on the quiz?

17. Jason has twice as many red marbles as blue marbles. He puts them in two jars in such a way that the ratio of the number of red marbles to blue marbles in jar I is 2:7 and there are only red marbles in jar II. The number of red marbles in jar II is how many times the number of red marbles in jar I?

18. If a and b are positive integers and their product is 3 times their sum, what is the value of 123

S T O P

YOU MAY GO BACK AND REVIEW THIS SECTION IN THE REMAINING TIME, BUT DO NOT WORK IN ANY OTHER SECTION UNTIL TOLD TO DO SO.

SECTION 8

Time—20 Minutes 19 Questions

Select the best answer to each of the following questions; then blacken the appropriate space on your answer sheet.

Each of the following sentences contains one or two blanks; each blank indicates that a word or set of words has been left out. Below the sentence are five words or phrases, lettered A through E. Select the word or set of words that best completes the sentence.

Example:

Fame is ----; today’s rising star is all too soon tomorrow’s washed-up has-been.

(A) rewarding

(B) gradual

(C) essential

(D) spontaneous

(E) transitory

123

  1. Famous in her time and then forgotten, the seventeenth-century Dutch painter Judith Leyster was ---- obscurity when, in 1993, the Worcester Art Museum organized the first retrospective exhibition of her work.

(A) resigned to

(B) rewarded with

(C) rescued from

(D) indifferent to

(E) worthy of

  2. The testimony of eyewitnesses is notoriously --; emotion and excitement all too often cause our minds to distort what we see.

(A) judicious

(B) interdependent

(C) credible

(D) unreliable

(E) gratifying

  3. During the Dark Ages, hermits and other religious ---- fled the world to devote themselves to silent contemplation.

(A) renegades

(B) skeptics

(C) altruists

(D) recluses

(E) convictions

  4. No real-life hero of ancient or modern days can surpass James Bond with his nonchalant ---- of death and the ---- with which he bears torture.

(A) contempt..distress

(B) disregard..fortitude

(C) veneration..guile

(D) concept..terror

(E) ignorance..fickleness

  5. A code of ethics governing the behavior of physicians during epidemics did not exist until 1846 when it was ---- by the American Medical Association.

(A) rescinded

(B) promulgated

(C) presupposed

(D) depreciated

(E) implied

  6. Unlike the highly ---- Romantic poets of the previous century, Arnold and his fellow Victorian poets were ---- and interested in moralizing.

(A) rhapsodic..lyrical

(B) frenetic..distraught

(C) emotional..didactic

(D) sensitive..strange

(E) dramatic..warped

The questions that follow the next two passages relate to the content of both, and to their relationship. The correct response may be stated outright in the passage or merely suggested.

Questions 7–19 are based on the following passages.

The following passages deal with the exotic world of subatomic physics. Passage 1, written by a popularizer of contemporary physics, was published in 1985. Passage 2 was written nearly 15 years later.

Passage 1

The classical idea of matter was something
with solidity and mass, like wet stone dust
pressed in a fist. If matter was composed of
atoms, then the atoms too must have solidity and

Line (5) mass. At the beginning of the twentieth century
the atom was imagined as a tiny billiard ball or a
granite pebble writ small. Then, in the physics of
Niels Bohr, the miniature billiard ball became
something akin to a musical instrument, a finely

(10) tuned Stradivarius 10 billion times smaller than
the real thing. With the advent of quantum
mechanics, the musical instrument gave way to
pure music. On the atomic scale, the solidity and
mass of matter dissolved into something light and

(15) airy. Suddenly physicists were describing atoms
in the vocabulary of the composer—“resonance,”
“frequency,” “harmony,” “scale.” Atomic electrons
sang in choirs like seraphim, cherubim,
thrones, and dominions. Classical distinctions

(20) between matter and light became muddled. In the
new physics, light bounced about like particles,
and matter undulated in waves like light.

In recent decades, physicists have uncovered
elegant subatomic structures in the music of

(25) matter. They use a strange new language to describe
the subatomic world: quark, squark, gluon,
gauge, technicolor, flavor, strangeness, charm
.
There are up quarks and down quarks, top quarks
and bottom quarks. There are particles with truth

(30) and antitruth, and there are particles with naked beauty.
The simplest of the constituents of ordinary
matter—the proton, for instance—has taken
on the character of a Bach fugue, a four-part
counterpoint of matter, energy, space, and time.

(35) At matter’s heart there are arpeggios, chromatics,
syncopation. On the lowest rung of the chain of
being, Creation dances.

Already, the astronomers and the particle
physicists are engaged in a vigorous dialogue.

(40) The astronomers are prepared to recognize that
the large-scale structure of the universe may have
been determined by subtle interactions of particles
in the first moments of the Big Bang. And the
particle physicists are hoping to find confirmation

(45) of their theories of subatomic structure in the
astronomers’ observations of deep space and time.
The snake has bitten its tail and won’t let go.

Passage 2

Consider a dew drop, poised at the tip of a
grass blade. Only one millimeter in diameter, this

(50) tiny dew drop is composed of a billion trillion
molecules of water, each consisting of two hydrogen
atoms and one oxygen atom (H2O). At the
onset of the twentieth century, this was the
accepted view of the nature of matter. Atoms

(55) were seen as matter’s basic building blocks,
elementary or fundamental particles that could not
be divided into anything smaller.

This relatively simple picture, however,
changed drastically as physicists came to explore

(60) the secrets of the subatomic world. The once-indivisible
atom, split, was revealed to consist of a
nucleus made up of protons and neutrons around
which electrons orbited. Protons and neutrons, in
turn, were composed of even smaller subatomic

(65) particles whimsically dubbed quarks. At first, theorists
claimed that all matter was made of three
fundamental particles: electrons and paired up and
down quarks. Later, however, experiments with
powerful accelerators and colliding particle beams

(70) suggested the existence of other pairs of quarks,
three generations in all, whose mass increased
with each generation. Lightest of all were the first
generation quarks, up and down, which combined
to create the basic protons and neutrons; somewhat

(75) heavier were the second generation quarks,
strange and charm, the building blocks of the
more esoteric particles produced in the physicists’
labs. Then in 1977 a team headed by Fermilab
physicist Leon Lederman uncovered the possibility

(80) of a third generation of quarks. Using new
accelerators with higher energies, they produced
a short-lived heavy particle, the upsilon, whose
properties suggested it could not be made of the
four quarks then known. They concluded it must

(85) be made of a fifth quark, which they named bottom,
whereupon scientists throughout the world
set off in hot pursuit of bottom’s hypothetical
partner, top.

The hunt for the top quark consumed the

(90) world’s particle physicists for nearly twenty
years. It was their Grail, and they were as
determined as any knight of King Arthur’s court to
succeed in their holy quest. To Harvard theorist
Sheldon Glashow in 1994, it was “not just

(95) another quark. It’s the last blessed one, and the
sooner we find it, the better everyone will feel.”
Indeed, they had to find it, for the Standard
Model of particle physics, the theoretical synthesis
that reduced the once-maddening hordes of

(100) particles (the so-called “particle zoo”) to just a
few primary components, hinged upon its existence.
Physicists likened the missing quark to the
keystone of an arch: the Standard Model, like an
arch, was supported by all its constituents, but it

(105) was the keystone, the last piece to go in, that
ensured the structure’s stability.

In 1995 the physicists found the keystone to
their arch, and with it, new questions to answer.
Surprisingly the top quark was far heavier than

(110) theorists had predicted, nearly twice as heavy
in fact. Fermilab physicist Alvin Tollestrup originally
had estimated top to weigh at least as much
as a silver atom. At the hunt’s end, top was determined
to have a mass similar to that of an atom of

(115) gold. (With an atomic weight of 197, a gold atom
is made up of hundreds of up and down quarks.)
The question thus remains, why is top so massive?
Why does any fundamental particle have
mass? With its astonishing heft, the top quark

(120) should help clarify the hidden mechanisms that
make some particles massive while others have
no mass at all.

  7. The author of Passage 1 refers to quarks, squarks, and charms (paragraph 2) primarily in order

(A) demonstrate the similarity between these particles and earlier images of the atom

(B) make a distinction between appropriate and inappropriate terms

(C) object to suggestions of similar frivolous names

(D) provide examples of idiosyncratic nomenclature in contemporary physics

(E) cite preliminary experimental evidence supporting the existence of subatomic matter

  8. The author’s tone in the second paragraph of Passage 1 can best be described as one of

(A) scientific detachment

(B) moderate indignation

(C) marked derision

(D) admiring wonder

(E) qualified skepticism

9. “Matter’s heart” mentioned in line 35 is

(A) outer space

(B) the subatomic world

(C) the language of particle physics

(D) harmonic theory

(E) flesh and blood

10. The final paragraph of Passage 1 indicates that the author regards the talks between the astronomers and the particle physicists as

(A) contentious

(B) unrealistic

(C) spirited

(D) distracting

(E) poetic

11. In line 47, the image of the snake biting its tail is used to emphasize

(A) the dangers of circular reasoning

(B) the vigor inherent in modern scientific dialogue

(C) the eventual triumph of the classical idea of matter

(D) the unity underlying the astronomers’ and particle physicists’ theories

(E) the ability of contemporary scientific doctrine to swallow earlier theories

12. In line 83, “properties” most nearly means

(A) lands

(B) titles

(C) investments

(D) civilities

(E) characteristics

13. Glashow’s comment in lines 94–96 reflects his

(A) apprehension

(B) impatience

(C) imagination

(D) jubilation

(E) spirituality

14. The references to the “keystone” of an arch (lines 103 and 105) serve to

(A) diminish the top quark’s status to that of a commodity

(B) provide an accurate physical description of the elusive particle

(C) highlight the contrast between appearance and reality

(D) give an approximation of the top quark’s actual mass

(E) illustrate the importance of the top quark to subatomic theory

15. In line 101, “hinged” most nearly means

(A) folded

(B) vanished

(C) remarked

(D) depended

(E) weighed

16. The author of Passage 2 does all of the following EXCEPT

(A) cite an authority

(B) use a simile

(C) define a term

(D) pose a question

(E) deny a possibility

17. The author of Passage 2 mentions the gold atom (lines 114 and 115) primarily to

(A) clarify the monetary value of the top quark

(B) explain what is meant by atomic weight

(C) illustrate how hefty a top quark is compared to other particles

(D) suggest the sorts of elements studied in highenergy accelerators

(E) demonstrate the malleability of gold as an element

18. As Passage 2 suggests, since the time Passage 1 was written, the Standard Model has

(A) determined even more whimsical names for the subatomic particles under discussion

(B) taken into account the confusion of the particle physicists

(C) found theoretical validation through recent experiments

(D) refuted significant aspects of the Big Bang theory of the formation of the universe

(E) collapsed for lack of proof of the existence of top quarks

19. The author of Passage 2 would most likely react to the characterization of the constituents of matter in lines 31–37 by pointing out that

(A) this characterization has been refuted by prominent physicists

(B) the characterization is too fanciful to be worthwhile

(C) the most recent data on subatomic particles support this characterization

(D) this characterization supersedes the so-called Standard Model

(E) the current theoretical synthesis is founded on this characterization

S T O P

YOU MAY GO BACK AND REVIEW THIS SECTION IN THE REMAINING TIME, BUT DO NOT WORK IN ANY OTHER SECTION UNTIL TOLD TO DO SO.

SECTION 9

Time—20 Minutes 16 Questions

For each problem in this section determine which of the five choices is correct and blacken the corresponding choice on your answer sheet. You may use any blank space on the page for your work.

Notes:

• You may use a calculator whenever you think it will be helpful.

• Only real numbers are used. No question or answer on this test involves a complex or imaginary number.

• Use the diagrams provided to help you solve the problems. Unless you see the words “Note: Figure not drawn to scale” under a diagram, it has been drawn as accurately as possible. Unless it is stated that a figure is three-dimensional, you may assume it lies in a plane.

• For any function f, the domain, unless specifically restricted, is the set of all real numbers for which f(x) is also a real number.

Reference Information

123

123

  1. If 123 then a =

(A) 123

(B)  123

(C)  123

(D)  3

(E)  4

  2. What is the value of 2x2 – 3x – 7 when x = –5?

(A)   28

(B)   42

(C)   58

(D)   78

(E) 108

  3. What is the diameter of a circle whose area is A?

(A) 123

(B) 123

(C) 123

(D) 123

(E) 123

Questions 4 and 5 refer to the following bar graph.

The graph shows the number of books read in January 2010 by the five members of a book club.

123

  4. What was the total number of books read in January 2010 by the members of the club?

(A) 14

(B) 15

(C) 16

(D) 17

(E) 18

  5. What percent of the members read more books than the average (arithmetic mean) number of books read?

(A) 20%

(B) 40%

(C) 50%

(D) 60%

(E) 80%

  6. Laurie inherited 40% of her father’s estate. After paying a tax equal to 30% of her inheritance, what percent of her father’s estate did she own?

(A) 10%

(B) 12%

(C) 25%

(D) 28%

(E) 30%

  7. If it is now September, what month will it be 555 months from now?

(A) April

(B) June

(C) September

(D) November

(E) December

123

  8. The graph of y = f (x) is shown in the figure above. Which of the following is a point on the graph of y = –f (x)?

(A) (0, –4)

(B) (0, 2)

(C) (–1, 1)

(D) (–1, –1)

(E) (2, 0)

  9. What is the value of a if a is positive and a 123 a 123 a = a + a + a?

(A) 123

(B) 123

(C) 3

(D) 3 123

(E) 9

10. What is the volume, in cubic inches, of a cube whose surface area is 60 square inches?

(A) 10 123

(B) 15 123

(C) 60 123

(D) 1000

(E) 3375

11. If 123 what is the value of f (16)?

(A)   6

(B)   8

(C) 12

(D) 32

(E) 64

12. If the circumference of circle I is equal to the diameter of circle II, what is the ratio of the area of circle II to the area of circle I?

(A) 123

(B) 123

(C) 123

(D) 1232

(E) 41232

123

13. The dartboard shown above is divided into 6 regions, all the same size. If a dart lands randomly on the board, what is the probability that it lands on a prime number?

(A) 123

(B) 123

(C) 123

(D) 123

(E) 123

14. If A is the set of integers between –50 and 50, and a number is in set B if it is the cube of a number in set A, how many elements of set B are in set A?

(A)     2

(B)     6

(C)     7

(D)   11

(E) 101

15. If A is 25 kilometers east of B, which is 12 kilometers south of C, which is 9 kilometers west of D, how far, in kilometers, is A from D?

(A) 20

(B) 5 123

(C) 5 123

(D) 10 123

(E) 71

16. If ab, and c are positive numbers such that 3a = 4b = 5c, and if a + b = kc, what is the value of k?

(A) 123

(B) 123

(C) 123

(D) 123

(E) 123

S T O P

YOU MAY GO BACK AND REVIEW THIS SECTION IN THE REMAINING TIME, BUT DO NOT WORK IN ANY OTHER SECTION UNTIL TOLD TO DO SO.

SECTION 10

Time—10 Minutes 14 Questions

For each of the following questions, select the best answer from the choices provided and fill in the appropriate circle on the answer sheet.

Some or all parts of the following sentences are underlined. The first answer choice, (A), simply repeats the underlined part of the sentence. The other four choices present four alternative ways to phrase the underlined part. Select the answer that produces the most effective sentence, one that is clear and exact, and blacken the appropriate space on your answer sheet. In selecting your choice, be sure that it is standard written English, and that it expresses the meaning of the original sentence.

Example:

The first biography of author Eudora Welty came out in 1998 and she was 89 years old at the time.

(A) and she was 89 years old at the time

(B) at the time when she was 89

(C) upon becoming an 89 year old

(D) when she was 89

(E) at the age of 89 years old

123

  1. Into her shopping basket she placed her favorite vegetables, an assortment of fresh fruit, and she included a loaf of French bread.

(A) and she included a loaf of French bread

(B) and a loaf of French bread

(C) and she also included a loaf of French bread

(D) a loaf of French bread as well

(E) and she includes a loaf of French bread

  2. Heather Hurst’s paintings and architectural renderings of the pre-Columbian Americas not only recover records that were previously lost, but these are works of art in their own right.

(A) not only recover records that were previously lost, but these are works of art

(B) not only recover records that had been previously lost, but these are works of art

(C) not only recover previously lost records but also are works of art

(D) do not recover only records that were previously lost, but these are works of art

(E) not only recovers records that were previously lost, but they are works of art

  3. Today, among twentieth-century artists, Salvador Dali’s renown is probably exceeded only by Picasso.

(A) artists, Salvador Dali’s renown is probably exceeded only by Picasso

(B) artists, Salvador Dali is probably exceeded in renown only by Picasso’s

(C) artists, Salvador Dali’s renown is probably exceeded only by Picasso’s

(D) artists, Salvador Dali is only exceeded in renown probably by only Picasso

(E) artists, Salvador Dali’s renown is only probably exceeded by Picasso’s

  4. So many of the internal workings of the lungs change at night that lung diseases, particularly asthma, has become the best studied of the nighttime illnesses.

(A) asthma, has become the best studied of the nighttime illnesses

(B) asthma, has become the best studied nighttime illnesses

(C) asthma, has become the better studied of the nighttime illnesses

(D) asthma, have become the best studied of the nighttime illnesses

(E) asthma, have been becoming the better studied out of all the nighttime illness

  5. There are a long list of causes of air pollution, ranging from automobile exhaust to methane emissions from livestock.

(A) There are a long list of causes of air pollution,

(B) There were a long list of things causing air pollution,

(C) There are a lengthy list of causes of air pollution,

(D) There have been a long list of causes of air pollution,

(E) There is a long list of causes of air pollution,

  6. Acupuncture has been widely used for years to ease chronic pain conditions, studies have repeatedly endorsed its usefulness.

(A) Acupuncture has been widely used for years to ease chronic pain conditions, studies

(B) Although acupuncture having been been widely used for years to ease chronic pain conditions, studies

(C) Acupuncture has been widely used for years to ease chronic pain conditions, and studies

(D) Due to the fact that acupuncture has been widely used for years to ease chronic pain conditions, studies

(E) Because acupuncture has been widely used for years to ease chronic pain conditions is the reason why studies

  7. Lower Manhattan was a seasonal home for the Lenni Lenape Indians, who granted the Dutch settlers land-use rights to Manhattan, but did not actually sell it for $24 in trinkets.

(A) Indians, who granted the Dutch settlers landuse rights to Manhattan, but

(B) Indians, which granted the Dutch settlers landuse rights to Manhattan, but

(C) Indians, who granted the Dutch settlers landuse rights to Manhattan, however they

(D) Indians, and they granted the Dutch settlers land-use rights to Manhattan, but

(E) Indians, where they granted the Dutch settlers land-use rights to Manhattan; but they

  8. From papayas in Hawaii to canola in Canada, the spread of pollen or seeds from genetically engineered plants are evolving from an abstract scientific worry into a significant practical problem.

(A) plants are evolving from an abstract scientific worry into

(B) plants are evolving from an abstractly scientific worry into

(C) plants are in process of evolving from an abstract scientific worry into

(D) plants is evolving from an abstract scientific worry into

(E) plants having evolved from an abstract scientific worry into

  9. After removing their skins, the children sliced the carrots into sticks for dipping.

(A) After removing their skins,

(B) After they removed their skins,

(C) After they had removed their skins,

(D) After removing the carrots’ skins,

(E) After they had removed the skins from the carrots,

10. Opinion polls show the public has about as dim a view of pharmaceutical companies as tobacco companies.

(A) has about as dim a view of pharmaceutical companies as tobacco companies

(B) have about as dim a view of pharmaceutical companies as tobacco companies

(C) has about as dim a view of pharmaceutical companies as it does of tobacco companies

(D) has almost so dim a view of pharmaceutical companies as of tobacco companies

(E) has approximately as dim a view of pharmaceutical companies as tobacco companies

11. The adjacent homes were dissimilar enough to justify their radically different prices.

(A) to justify their radically different prices

(B) to justify its radically different prices

(C) to be justified by their radically different prices

(D) to justify there radically different prices

(E) to be a justification for their radically different prices

12. The pale white petals of the gardenia possess a scent of great sweetness and subtlety and the scent has intrigued many perfume-makers.

(A) subtlety and the scent has

(B) subtlety, that being the reason why the scent has

(C) subtlety, but the scent has

(D) subtlety, a scent that has

(E) subtlety, it has

13. Attempting to maximize the income-producing potential of her pension plan by investing a substantial amount in so-called junk bonds.

(A) Attempting to maximize the income-producing potential of her pension plan by investing a substantial amount

(B) Attempting to maximize the income-producing potential of her pension plan by substantially investing an amount

(C) She made an attempt to produce the maximum potentiality in income out of her pension plan and she invested a substantial amount

(D) In an attempt to produce the maximum income-producing potential from her pension plan by investing a substantial amount

(E) She attempted to maximize the incomeproducing potential of her pension plan by investing a substantial amount

14. Seldom do the barriers between the races seem less in evidence than on this league-leading high school football team.

(A) Seldom do the barriers between the races seem less in evidence than on this league-leading high school football team

(B) More so than on other teams, they seem to be less evident barriers between the races on this league-leading high school football team

(C) On this league-leading high school football team, more so than on other teams, the barriers between the races are less in evidence, it seems

(D) The barriers between the races do seem fewer in evidence seldom on this league-leading high school football team

(E) Seldom less than on this league-leading high school football team does the barriers between the races seem less in evidence

S T O P

YOU MAY GO BACK AND REVIEW THIS SECTION IN THE REMAINING TIME, BUT DO NOT WORK IN ANY OTHER SECTION UNTIL TOLD TO DO SO.

Answer Key

Note: The letters in brackets following the Mathematical Reasoning answers refer to the sections of Chapter 9 in which you can find the information you need to answer the questions. For example, 1. C [E] means that the answer to question 1 is C, and that the solution requires information found in Section 9-E: Averages.

Section 2      Critical Reading

  1. C

  2. C

  3. D

  4. E

  5. D

  6. E

  7. C

  8. B

  9. D

10. E

11. E

12. C

13. E

14. A

15. B

16. B

17. A

18. E

19. C

20. A

21. E

22. B

23. D

24. C

Section 3      Mathematical Reasoning

  1. A [G]

  2. D [B]

  3. D [I]

  4. D [Q, E]

  5. A [Q, C]

  6. B [C]

  7. B [A, O]

  8. C [E, G]

  9. D [J, K, L]

10. D [A]

11. E [B, M]

12. B [J]

13. C [A, G]

14. A [E]

15. A [G]

16. B [H]

17. E [J]

18. A [D]

19. A [F]

20. B [L]

Section 4      Writing Skills

  1. C

  2. D

  3. C

  4. A

  5. D

  6. A

  7. C

  8. A

  9. D

10. E

11. B

12. D

13. C

14. E

15. D

16. E

17. C

18. B

19. C

20. D

21. C

22. B

23. D

24. A

25. C

26. C

27. C

28. E

29. C

30. C

31. B

32. D

33. D

34. E

35. C

Section 5

On this test, Section 5 was the experimental section. It could have been an extra critical reading, mathematics, or writing skills section. Remember: on the SAT you take, the experimental section may be any section from 2 to 7.

Section 6      Critical Reading

  1. D

  2. C

  3. E

  4. A

  5. B

  6. E

  7. D

  8. B

  9. B

10. C

11. B

12. D

13. D

14. A

15. C

16. C

17. D

18. E

19. B

20. E

21. C

22. E

23. D

24. A

Section 7      Mathematical Reasoning

Multiple-Choice Questions

  1. C [G]

  2. C [D]

  3. D [G]

  4. E [L, N]

  5. E [C, D]

  6. C [R]

  7. B [F]

  8. C [P]

Grid-in Questions

123

123

Section 8      Critical Reading

  1. C

  2. D

  3. D

  4. B

  5. B

  6. C

  7. C

  8. D

  9. D

10. B

11. D

12. E

13. B

14. E

15. D

16. E

17. C

18. C

19. B

Section 9      Mathematical Reasoning

  1. B [B, G]

  2. C [A, F]

  3. A [L]

  4. E [Q]

  5. D [C, E, Q]

  6. D [C]

  7. E [R]

  8. E [R]

  9. B [A]

10. A [M]

11. A [A, R]

12. D [D, L]

13. C [A, O]

14. C [A]

15. A [J, K]

16. E [G]

Section 10      Writing Skills

  1. B

  2. C

  3. C

  4. D

  5. E

  6. C

  7. A

  8. D

  9. D

10. C

11. A

12. D

13. E

14. A

Score Your Own SAT Essay

Use this table as you rate your performance on the essay-writing section of this Model Test. Circle the phrase that most accurately describes your work. Enter the numbers in the scoring chart below. Add the numbers together and divide by 6 to determine your total score. The higher your total score, the better you are likely to do on the essay section of the SAT.

Note that on the actual SAT two readers will rate your essay; your essay score will be the sum of their two ratings and could range from 12 (highest) to 2 (lowest). Also, they will grade your essay holistically, rating it on the basis of their overall impression of its effectiveness. They will not analyze it piece by piece, giving separate grades for grammar, vocabulary level, and so on. Therefore, you cannot expect the score you give yourself on this Model Test to predict your eventual score on the SAT with any great degree of accuracy. Use this scoring guide instead to help you assess your writing strengths and weaknesses, so that you can decide which areas to focus on as you prepare for the SAT.

Like most people, you may find it difficult to rate your own writing objectively. Ask a teacher or fellow student to score your essay as well. With his or her help you should gain added insights into writing your 25-minute essay.

123

Self-Scoring Chart

Scoring Chart (Second Reader)

For each of the following categories,
rate the essay from 1 (lowest)
to 6 (highest)

For each of the following categories,
rate the essay from 1 (lowest)
to 6 (highest)

Position on the Topic

123

Position on the Topic

123

Organization of Evidence

123

Organization of Evidence

123

Sentence Structure

123

Sentence Structure

123

Level of Vocabulary

123

Level of Vocabulary

123

Grammar and Usage

123

Grammar and Usage

123

Overall Effect

123

Overall Effect

123

TOTAL

123

TOTAL

123

(To get a score, divide the total by 6)

123

(To get a score, divide the total by 6)

123

Calculate Your Raw Score

Critical Reading

123

Mathematical Reasoning

123

Writing Skills

123

Evaluate Your Performance

123

Identify Your Weaknesses

123

Identify Your Weaknesses

123

Identify Your Weaknesses

123

ANSWERS EXPLAINED

Section 2 Critical Reading

  1. C.    If one is alert and insightful, one’s faculties (mental powers) are intact (sound or whole). Note how the phrase set off by the comma restates and clarifies the idea that Picasso has continued to be perceptive and alert.

(Contrast Signal)

  2. C.    While suggests a contrast between the fates of the two dance forms during the 1940s. The decade was most noted for the growth of Black modern dance. However, it was also noteworthy or significant for the decline of tap dancing.

(Contrast Signal)

  3. D.    Something beneficial or helpful in small amounts may be toxic (poisonous) in large amounts.

Remember to watch for signal words that link one part of the sentence to another. The use of “though” in the second clause sets up a contrast. The missing word must be an antonym or near-antonym for beneficial.

(Argument Pattern)

  4. E.    The encroachments or trespassing of human beings on the hawk’s territory would frighten the birds, leading them to shun or avoid their usual locations for breeding. Frightened away from their nests, disturbed in their breeding routines, the hawks would have fewer offspring. Thus, their numbers would diminish or dwindle. You can immediately eliminate Choices A, B, and C. Choices A and C you can rule out on the basis of general knowledge: when humans come close, wild birds abandon their nests (and their eggs): they have fewer offspring. Choice B you can rule out on the basis of usage. People may extrapolate or make projections on the basis of known data about the number of hawks. The “number of black hawks,” however, doesn’t extrapolate anything.

(Argument Pattern)

  5. D.    To aim at a mass market is to try to appeal to the lowest common denominator. British films, which rarely aim at the masses, instead try to appeal to the elite. They are thus elitist.

(Contrast Signal)

  6. E.    The key phrase here is “worth the trouble.” What sort of person creates trouble for his employers? Not a congenial (agreeable) or popular one. You can immediately eliminate Choices B and C. A cantankerous(badtempered) employee creates problems. However, if he turns in meticulous (very careful and exact) work, his employers may think he’s worth the trouble he makes. Note that, after eliminating the answer choices whose first word does not work in the sentence, you must check the second words of the remaining answer choices. A domineering (bossy) or fastidious (fussy) employee might create problems around the newspaper office. However, he would not get on his employers’ good side by turning in wearisome (boring) or garbled (confused) stories.

(Argument)

  7. C.    Because the opossum is impervious to (unharmed by) the poison, it can treat the rattlesnake as a potential source of food and not as a lethal or deadly enemy. Note the cause and effect signal thus. The nature of the opossum’s response to the venom explains why it can look on a dangerous snake as an easy prey.

(Cause and Effect Signal)

  8. B.    By definition, someone who breaks with established convention is iconoclastic or nonconformist. Go through the answer choices, eliminating those you can. Choices A and E are incorrect. Someone who departs from tradition is unlikely to be derivative (lacking originality) or trite (commonplace; timeworn). Choices C and D are incorrect. Someone who infuriates (enrages) the traditionalists is controversial, not uncontroversial, and is unlikely to be venerated (deeply respected) by them. This is one of the last sentence completion questions, so its answer is an extremely difficult word.

(Definition Pattern)

  9. D.    The “peculiar” advantage that a best-selling novel has over an original script is its particular, special advantage: the story’s popularity with a substantial segment of the population is guaranteed.

10. E.    The final sentence of the passage explains the difficulty involved in working with best-selling novels: filmmakers have to pay so much money to get the screen rights that the producers often wind up losing money on the movie. In other words, the problem is the financial impact of purchasing rights to adapt the novel.

11. E.    The pitch is the steep slope of the rapids into which Thornton has fallen.

12. C.    With its forceful descriptive phrases (“suck of the water,” “last steep pitch, “scraped... bruised... struck”) and its sharp exclamations, the passage has a strongly urgent tone.

13. E.    Substitute the answer choices in the original sentence. The sergeant is a person who might have been a deputy sheriff before he joined the army—that is, in his civil or nonmilitary life.

14. A.    Paragraph 1 presents a general picture of the man on the bridge, the executioners and the officer standing nearby, the sentinels at the far ends of the bridge. Cinematically, it is like a wide-angle shot of the whole panorama. Paragraph 2 takes a closer look at the man, examining his clothes, his face, his expression. It is as if the camera has moved in for a close-up shot.

15. B.    The author’s comment that the man “had a kindly expression that one would hardly have expected in one whose neck was in the hemp” suggests that he is an unlikely candidate for execution and that some unusual circumstances must have brought him to this fate.

16. B.    In calling the military code “liberal” because it doesn’t exclude members of the upper classes from being executed, the author is being highly ironic. Generally, people would like regulations to be interpreted liberally to permit them to do the things they want. Here, the liberal military code is permitting the man to be hanged. Clearly, the gentleman facing execution would have preferred the code to be less liberal in this case.

17. A.    Farquhar agrees readily with the saying that all is fair in love and war. This implies he is willing to use underhanded or unfair methods to support his [the Southern] cause.

18. E.    Farquhar has no objection to performing humble errands or undertaking dangerous tasks as long as these tasks are appropriate to someone who sees himself as a sort of “undercover soldier,” a secret agent of the Confederacy. Anything he does must be consistent or compatible with his image of himself in this role.

19. C.    At heart a soldier, Farquhar fundamentally agrees that all’s fair in war. He doesn’t particularly qualify or restrict his commitment to this viewpoint: he’s ready to go out and do something underhanded for his cause without much restriction as to what he’s willing to do.

20. A.    Mrs. Farquhar’s readiness to fetch water for the gray-clad Confederate soldier suggests some degree of sympathy on her part for the Confederate cause. Choices B and D are incorrect. There is nothing in the passage to suggest either of them. Choices C and E are incorrect. Mrs. Farquhar’s action, in hospitably fetching water “with her own white hands,” contradicts them.

21. E.    Earlier in the passage, Farquhar is described as frustrated by “the inglorious restraint” preventing his serving the Southern cause. He sees the life of the soldier as larger than that of the civilian, a life filled with opportunities for distinction, for renown. Thus, when he speaks about someone managing to sneak past the guards and accomplishing something for the cause, he is envisioning himself as a hero.

22. B.    Farquhar wishes to prevent the Yankee advance. To do so, he must somehow damage the railroad, its bridges, its tunnels, or its trains. The soldier tells him that some highly flammable driftwood is piled up at the base of the wooden railroad bridge. Clearly, it would make sense for Farquhar to try to set fire to the driftwood in an attempt to burn down the bridge.

23. D.    The phrase “burn like tow” and the reference to dry driftwood suggest that tow is highly combustible. Remember: when asked to give the meaning of an unfamiliar word, to look for nearby context clues.

24. C.    The scout is a Yankee soldier disguised as a member of the enemy. By coming to the Farquhars’ plantation in Confederate disguise, he is able to learn they are sympathetic to the enemy. By telling Farquhar of the work on the bridge, stressing both the lack of guards and the abundance of fuel, he is tempting Farquhar into an attack on the bridge (and into an ambush). The scout’s job is to locate potential enemies and draw them out from cover.

Section 3 Mathematical Reasoning

In each mathematics section, for many problems, an alternative solution, indicated by two asterisks(**), follows the first solution. When this occurs, one of the solutions is the direct mathematical one and the other is based on one of the tactics discussed in Chapter 8 or 9.

  1. A.    If 2x + 4x + 6x = –12, then 12x = –12 and x = –1.

**Note that x must be negative, so only A and B are possible. Test these choices.

  2. D. 123    Use your calculator: 1.1 123 1.9 = 2.09, which, to the nearest tenth, is 2.1.

  3. D.    Since ABCD is a square, y = 90. Then x + 90 + 40 = 180 and so x = 50.

123

Then x = z = u = w = 50 [When parallel lines are cut by a transversal, the four acute angles have the same measure (KEY FACT I6)].

**Use TACTIC 2: trust the diagram; w appears to be slightly more than a 45° angle.

  4. D.    The average is just the sum of the number of students in the five classes (125) divided by 5: 125 ÷ 5 = 25.

  5. A.    In class A, one-fourth, or 25% (5 of 20), of the students are in the band. In each of the other classes, the number in the band is less than one-fourth of the class.

  6. B. 123    Since 50% of the trip was completed on Monday and 10% of the trip is left, the 100 kilometers traveled on Tuesday represents the other 40% of the total distance, d, so

0.40d = 100 123 d = 100 ÷ 0.40 = 250.

**Estimate. Since half of the trip was completed Monday, the 100 kilometers traveled on Tuesday plus the 10% still to go constitutes the other half. The 100 kilometers by itself is slightly less than half, and 200 kilometers would be slightly less than the whole distance. Of the choices, only 250 is possible.

  7. B.    There are nine positive integers less than 10: 1, 2, …, 9. For which of them is 123 Only 1 and 123 and 123 When 123 and for all the others 123 The probability is 123

  8. C.    Let n represent the number of members of the club before Jean joined. These members raised a total of 85n dollars (KEY FACT E1). After Jean was in the club, the total raised was 85n + 50, the average was 80, and the

123

**Backsolve, starting with 6, Choice C. If there were 6 members, they would have raised 6 123 $85 = $510. After Jean joined and raised $50, there would have been 7 members who raised a total of $510 + $50 = $560. And $560 ÷ 7 = $80. Choice C works.

  9. D.    Draw pictures.

• RST could be the vertices of a right triangle. (I is true.)

• RST could not be the vertices of a square: if RS is a side of the square, then RT = 123 RS, so 123 (II is false.)

123

• RST could all lie on a circle. (III is true.) Statements I and III only are true.

10. D.    Since the game takes 1 hour, or 60 minutes, and there are always 5 men playing, there is a total of 5 123 60 = 300 man-minutes of playing time. If that amount of time is evenly divided among the 12 players, each one plays 300 ÷ 12 = 25 minutes.

**Estimate. If 5 men played the first 10 or 12 minutes, and 5 other men played the next 10 or 12 minutes, who would be left to play the rest of the game? Since 10 and 12 are much too small, eliminate A and B. If 5 men played for 30 minutes, and 5 other men played the next 30 minutes, the game would be over and 2 men wouldn’t have played at all. Since 30 is too large, eliminate E. The answer must be 24 or 25. Guess.

11. E.    When A ounces of water are removed from pitcher II, that pitcher will contain B – A ounces. Since its capacity is B, pitcher II will be 123 full.

**Use TACTIC 7: plug in easy-to-use numbers. Suppose pitcher II holds 10 ounces and pitcher I holds 3. Then, if 3 ounces are poured from pitcher II into pitcher I, pitcher II will have 7 ounces and be 123 full. Which of the choices equals 123 when B = 10 and A = 3? Only 123.

12. B.    In 123ABCw + x + 40 = 180 123 w + x = 140.

Similarly, in 123ADEy + z + 40 = 180 123 y + z = 140. Then w + x + y + z = 140 + 140 = 280.

**Use TACTIC 2: trust the diagram; wxy, and z appear to be about 100, 45, 60, and 80, respectively, for a total of 285. Your estimate may well be slightly more or less, but should surely be between 240 and 310. With anything less than 300, guess 280; if your estimate is over 300, you might pick 320.

123

13. C.    |x – 5| + 10 = 15 123 |x – 5| = 5 123
x – 5 = 5     or     x – 5 = –5 123
x = 10         or     x = 0

The greatest value is 10.

14. A.    Use TACTIC 17:

123

To get the average, divide the sum by 3:

123

(Note: You could solve for xyz, but you shouldn’t.)

15. A.    Let x = the number of passengers originally on the bus, and keep track of the comings and goings. At the first stop half the people got off, leaving 123x on the bus, and 1 more got on: 123x + 1. At the second stop 123 of the passengers got off, leaving two-thirds on the bus, and 1 person got on: 123 This simplifies to 123 which equals 15, so 123

**Work backwards. At the end there were 15 passengers, so before the last one got on, there were 14, which was 123 of the number before any got off at the second stop. At that point there were 21 passengers, meaning that, before 1 person got on at the first stop, there were 20, which is half of 40, the original number.

**Use TACTIC 5: backsolve. Start with C, 58: 29 got off and 1 got on; then there were 30; 10 got off and 1 got on; then there were 21, but we should have only 15. Since 58 is too big, eliminate C, D, and E. Try A or B; A works.

16. B.    Since the value of the house doubled every 6 years, in the past 30 years it has doubled 5 times. If the original price of the house was d dollars, the value today would be d(25) dollars.

25d = $320,000 12332d = $320,000, and so d = $10,000.

**You can just work backwards. Six years ago, the house was worth one half of $320,000, or $160,000. Six years earlier, it was worth $80,000. Continue dividing by 2: $320,000 123 $160,000 123 $80,000 123 $40,000 123$20,000 123 $10,000.

17. E.

123

Check each statement to see if it must be true.

• In 123ABC123 B measures 80°, so a + c = 80 and c = 80 – a. (I is true.)

• Since the measure of an exterior angle of a triangle equals the sum of the measures of the two opposite interior angles (KEY FACT J2), b = c + 50 123 c = b – 50. (II is true.)

• Since a + b = 130 and c + d = 130, then a + b = c + d. (III is true.)

Statements IIIand III are true.

18. A.    If c carpenters can build a garage in d days, then 1 carpenter will take c times as long, or cd days, and 2 cd days to build 2 garages. Finally, if the work is divided up among e carpenters, they will take 123 days.

123 **Use TACTIC 6: plug in easy-to-use numbers. If 2 carpenters can build a garage in 10 days, they will take 20 days to build 2 garages. It will take 4 carpenters half as long: 10 days. Which choice is equal to 10 when c= 2, d =10, and e = 4? Only 123. Remember: test each choice with your calculator, and eliminate a choice as soon as you can see that it is not equal to 10.

19. A.    This question is easier than it seems at first. In each fraction, the numerator is the negative of the denominator, so each fraction equals –1 and the sum of the fractions is –2.

**Of course, you can use TACTIC 6: plug in numbers. If a = 1 and b = 2, then each fraction is equal to –1.

20. B.    123

Section 4 Writing Skills

  1. C.    Error in modification. The opening phrase (“Unable to see more than three inches in front of her nose without corrective lenses”) describes Mary, not Mary’s search or Mary’s glasses.

  2. D.    The original sentence is both wordy and unidiomatic. Choice B is also wordy and changes the meaning of the original. Choice C introduces a comma splice error. Choice E introduces a sentence fragment after the semicolon. Only Choice D is an effective, idiomatic sentence.

  3. C.    The original sentence, while grammatically correct, is wordy. Choice C eliminates the wordiness without introducing new errors.

  4. A.    Sentence is correct. Choices B and C introduce errors in subject-verb agreement. Choices D and E are wordy and awkward.

  5. D.    Lack of parallelism. In Choice D, the phrase fascinating because of Trump’s shrewdness exactly parallels the earlier phrase infuriating because of Trump’s arrogance.

  6. A.    Sentence is correct. Choice B introduces an error in tense. Choice D introduces an error in subject-verb agreement (the verb should not be plural). Choices C and E are wholly confusing.

  7. C.    Shift of personal pronoun. Remember: everything in the sentence that has not been underlined is correct. Therefore, the pronoun you is correct; the pronoun one is incorrect. Choice C corrects the error in person economically.

  8. A.    Sentence is correct. Choices B and C introduce errors in subject-verb agreement and tense.

Choices D and E are wordy and awkward.

  9. D.    Error in pronoun-antecedent agreement. The antecedent is the plural noun communities; the appropriate pronoun, therefore, is their, not its.

10. E.    Errors in subject-verb agreement and in idiom. The subject is drop (singular); the verb should be singular as well. Replace have encouraged with has encouraged. Additionally, note that the buyers are encouraged to do something, namely, to apply for loans.

11. B.    The change in subordination emphasizes the built-in contrast between the bridge’s actual color and its name.

12. D.    Error in idiom. Use the preposition of with the adjective desirous: the shoppers are desirous of comparing prices.

13. C.    Error in subject-verb agreement. The subject is juxtaposition (singular); the verb should be singular as well. Replace reveal with reveals.

14. E.    Sentence is correct.

15. D.    Error in idiom. Use the preposition for with the adjective suitable: the amounts of water collected are rarely suitable for consumption without treatment.

16. E.    Sentence is correct.

17. C.    Lack of parallelism. Hunger and knack are both nouns. The third item in this series should also be a noun. Substitute a willingness for was willing.

18. B.    Adjective and adverb confusion. Change the adjective surprising to the adverb surprisingly, which then correctly modifies the adjective little.

19. C.    Error in subject-verb agreement. The verb should be needs to agree with the singular subject cream.

20. D.    Error in comparison. Only two events are being compared. Of the two, the Tevis Cup presents the greater challenge.

21. C.    Incorrect pronoun. The clause modifies movement, not writers. It was the movement that came to be known as the Harlem Renaissance.

22. B.    Error in logical comparison. The sentence is comparing Whitman’s poetry with Kipling’s poetry, not with Kipling the poet. The sentence should begin “Clearly, Whitman’s verses, unlike Kipling’s, are wholly unconventional.”

23. D.    Error in parallelism. Change an apprenticeship to to apprentice, paralleling to visit.

24. A.    Error in word usage. Perspective is a noun meaning viewpoint or vista; prospective is an adjective meaning expected or future. The sentence is discussing the proposed merger’s future or prospective advantages.

25. C.    Error in word usage. Latter, an adjective or noun, refers to the second of two persons or things; later, an adverb, refers to time. Initially (at first) the candidate raised funds via the Internet; later he tried other methods.

26. C.    Error in sequence of tenses. Change have been dozing to had been dozing.

27. C.    Error in tense. Replace would be with will be.

28. E.    Sentence is correct.

29. C.    The problem here involves a fusion of idioms. It is correct to say that the differences arise less from a dispute about the function of government than from a dispute about the nature of mankind. It would also be correct to say that the differences arise not from a dispute about the function of government but from a dispute about the nature of mankind. Do not mix the two constructions.

30. C.    All sentences except sentence 3 contribute to the paragraph’s main point, that celebrations help to unite people and keep traditions alive. Therefore, Choice C is the best answer.

31. B.    Choice A is grammatically correct, but the first clause is awkwardly expressed in the passive voice.

Choice B is clearly written and to the point. It is the best answer.

Choice C contains a dangling participle. The phrase Receiving new clothes should modify children, not money.

Choice D is awkwardly expressed.

Choice E is wordy and awkward.

32. D.    Choice A is fresh, but its tone is not consistent with the rest of the essay.

Choice B contains a comma splice between dinner and at.

Choice C emphasizes the idea properly, but lacks parallel construction.

Choice D places the emphasis where it belongs and expresses the idea effectively. It is the best answer.

Choice E is repetitious, and it contains an error in pronoun reference. The pronoun they has no specific antecedent.

33. D.    Choice A does not provide a significantly better transition.

Choice B does nothing to improve the relationship between paragraphs 3 and 4.

Choice C is awkwardly worded and does not include transitional material.

Choice D provides an effective transition between paragraphs. It is the best answer.

Choice E tries to provide a transition, but it is wordy and it contains a dangling participle.

34. E.    Choice A places emphasis on the location of the assassination instead of on the event itself, an emphasis that the writer did not intend.

Choice B contains a nonstandard usage: the phrase to remember about.

Choice C is grammatically correct but wordy.

Choice D is the same as B.

Choice E is a succinct and proper revision. It is the best answer.

35. C.    The main purpose of the last paragraph is to provide an example of a celebration that unites people and preserves tradition. Therefore, Choice C is the best answer.

Section 6 Critical Reading

  1. D.    The critics would regret any lapse on the part of a promising writer.

The adjective glowing is your clue that you are looking for a word with positive associations. Therefore, you can eliminate any word with negative ones. Choices A, B, and C have negative associations. Only Choice D or E can be correct.

(Cause and Effect Pattern)

  2. C.    A longing for old friends and familiar scenes is nostalgia or homesickness.

Remember: before you look at the choices, read the sentence and think of a word that makes sense.

Likely Words: homesickness, nostalgia, yearning.

(Definition)

  3. E.    Borrowers would complain that an old, appreciated borrowing policy had been set aside or superseded.

Remember: in double-blank sentences, go through the answer choices, testing the first words in each choice and eliminating those that don’t fit. The fact that the new policy has received complaints indicates that the old policy was viewed positively. You can immediately eliminate Choice B, disliked, and Choice D, ignored. Both are negative terms.

(Contrast Pattern)

  4. A.    Even though signals a contrast. The brain’s fundamental organization does not change.

However, the details of the brain’s organization do change: they remain plastic, pliable, capable of being molded or shaped.

(Contrast Signal)

  5. B.    The key phrase here is “in a few words.”

Although the movie is lavish in its beauty, it is not lavish in its use of words or film. Instead, it demonstrates economy of style.

(Definition)

  6. E.    The opening sentence of Passage 1 states that few, if any, people recalled Lady Mary’s efforts to fight smallpox. Her efforts have largely been forgotten. Likewise, the opening sentences of Passage 2 assert that some people “ignore the claims” of the African slave Onesimus, who played a small but important part in the battle against the deadly disease. Thus, both passages attempt to call attention to neglected historical figures.

  7. D.    Without her travels in the East, where she encountered the Eastern custom of inoculation, Lady Mary would not have been inspired to bring back this procedure to England. Thus, her smallpox-fighting efforts in England came about as a consequence of her travels in the East.

  8. B.    Not only did the Turks practice the custom of inoculation, they “even” held house parties so that inoculated youngsters could convalesce in company and in comfort. Clearly the procedure enjoyed widespread acceptance.

  9. B.    Both Montagu and Mather advocated inoculation, a foreign medical practice well known in Turkey and in parts of Africa.

10. C.    According to the author, “We have been taught to believe that our lives are better than the lives of those who came before us” and the lives of those today who live in similarly “primitive” circumstances. We base our belief that we Americans are well off today on the assumption that people in earlier generations and people now living in “primitive” circumstances have an inferior standard of living.

11. B.    The conventional wisdom is that the lives of primitive peoples are filled with toil. The author, however, states that primitives do little work. Thus, she regards the conventional wisdom with skepticism or doubt.

12. D.    According to the author, these “stone age peoples” have limited desires. They are not motivated by any particular desire for consumer goods or other material comforts.

13. D.    To raise an issue is to bring it up for discussion.

14. A.    Throughout the passage the author disputes the assumption made by the conventional wisdom that our economic progress has been an unmitigated blessing. She argues instead that we “have paid a price for prosperity.”

15. C.    The author makes an assertion: “We are eating more.” She then qualifies or limits her assertion: “but we are burning up those calories at work.” She repeats this pattern of assertion followed by qualification. She then draws her conclusion: it is hard to support the conventional wisdom that economic progress has been an unmixed blessing for us.

16. C.    The author provides the reader both with physical details of dress and bearing (appearances) and with comments about the motives and emotions (moral natures) of the Cardinal and Bosola.

Choice A is incorrect. The passage scarcely mentions the church. Choice B is incorrect.

The description of ecclesiastical costumes is only one item in the description of the Cardinal. Choice D is incorrect. While audiences today might well enjoy seeing the characters acted as described here, the author does not cite specific reasons why the play might appeal to modern audiences. Choice E is incorrect. The author’s purpose is to describe two separate roles, not to compare two interpretations of a single role.

17. D.    “Spare” is being used to describe the Cardinal’s physical appearance. He is tall and lean.

18. E.    The eagle is poised to strike “with exposed talons.” It, like the Cardinal, poises itself to strike with greater force. The imagery suggests the Cardinal’s mercilessness.

Choice A is incorrect. The Cardinal is not flighty (light-headed and irresponsible); he is cold and calculating. Choice B is incorrect. He loves power, not freedom. Choice C is incorrect. An eagle poised to strike with bare claws suggests violence, not eminence (fame and high position). Choice D is incorrect. Nothing in the passage suggests he is spiritual.

Beware of Eye-Catchers. “Eminence” is a title of honor applied to cardinals in the Roman Catholic church. Choice C may attract you for this reason.

19. B.    The Cardinal glories in his place in the hierarchy of the Church: his rank or status as an ecclesiastical lord.

20. E.    Although Bosola is not a “leather-jacketed” hoodlum, he is a hired “assassin,” a “hangman,” a mercenary killer (despite his scholarly taste and humanist disposition).

21. C.    Answer this question by the process of elimination.

Choice A is incorrect. In describing Bosola the author makes no use of rhetorical questions (questions asked solely to produce an effect).

Choice B is incorrect. Though the author makes many assertions about Bosola, he limits or qualifies many of them. For example, the author asserts that Bosola “is inwardly tormented.” He then immediately qualifies his assertion, adding “but not by undiluted passion.” Thus, the author does not chiefly use unqualified assertions in describing Bosola.

Choice D is incorrect. Dramatic irony is irony built in to a speech or a situation, which the audience understands, but which the characters onstage have yet to grasp. The author does not use this literary technique in describing Bosola.

Choice E is incorrect. The author makes one brief literary allusion (to Browning’s verse monologues). He does not chiefly use literary allusions in describing Bosola.

Only Choice C is left. It is the correct answer.

Throughout the passage’s final paragraph, the author describes Bosola through comparisons (“Like his black-and-white clothes,” “Like Duke Ferdinand”) and contrasts (“not . . . a leatherjacketed, heavy-booted tough,” “Still less . . . a sneering, leering, melodramatic villain”).

22. E.    The author is contrasting the two sides of Bosola, the scholar and the assassin. As a scholar, he is a man of perceptive intellect, noted for discrimination or discernment.

23. D.    Lines 61–66 state that Bosola “is inwardly tormented … (by) disgust at a world filled with knavery and folly, … in which he must play a part and that a lowly, despicable one.” The villainy and foolishness around him disgust him. He feels intellectually superior to the evil around him, yet must act the villain himself.

24. A.    The casual references to the elongated hands and features of El Greco’s work and to the trim beards and commanding stances in the work of Van Dyke imply that the author assumes the reader is familiar with both painters’ art.

Section 7 Mathematical Reasoning

MULTIPLE-CHOICE QUESTIONS

  1. C.    The left-hand side of (w + 12) – 12 = 12 is just w, so w = 12.

**Of course, you can use TACTIC 5: backsolve, starting (and ending) with C.

  2. C.    If 24 of the students are girls, then 40 – 24 = 16 are boys. The ratio of boys to girls is 16:24 which reduces to 2:3.

**Even if you can’t do the above, there are clearly fewer boys than girls, and so the ratio must be less than 1:1. Eliminate D and E and guess.

  3. D. 123    25 – 3 123 = 7 123 –3 123 = –18 123 123 = 6 123 x = 36.

**If you want to avoid the algebra, use TACTIC 5: backsolve. First eliminate choices A and B, since you can’t take the square root of a negative number. Then use your calculator to test the other choices. Only 36, choice D, works: 25 – 3123 = 25 – 3(6) = 25 – 18 = 7.

  4. E.    Since the formula for the area of a circle is A = 123 r2, to calculate the area you need to know the radius, which is just the distance from the center, O, to the point, P. Use the distance formula:

123 123

The area is 123

  5. E.    At the regular price, a CD costs d dollars, so at 50% off it costs 123 dollars. To find out how many you can buy, divide the amount of money, m, by the price per CD, 123:

123

123 **Use TACTIC 6: plug in easy-to-use numbers. If a CD regularly costs $10, then on sale at 50% off, they cost $5 each. How many can be purchased on sale for $20? The answer is 4. Which of the choices equals 4 when d = 10 and m = 20?

Only 123

  6. C.    123

  7. B.    123 123

**Use TACTIC 6: plug in a number for x. For example, if x = 3:

123 123

Only choice B is 10 when x = 3: 123

  8. C.    Sequence I: 2, 4, 6, 8, 10, … .
The nth term is 2n, so the 32nd term is 64.
Sequence II: 2, 4, 8, 16, 32, … .
The nth term is 2n, so the 32nd term is 232.
Finally, 123

GRID-IN QUESTIONS

  9. (7.5)    Here, 123ABC is a right triangle and its area is given by 123(AB)(BC). Since AB is vertical, find its length by subtracting the y-coordinates: AB = 4 – 1 = 3. Similarly, since BC is horizontal, find its length by subtracting the x-coordinates: BC = 6 – 1 = 5. Then area of 123ABC= 123(3)(5) = 123 = 7.5.

10. (98) 123    The easiest way is to simplify first: (a2 + b2) – (a2 – b2) = 2b2. Then 6 123 7 = 2(72) = 2(49) = 98.

**If you don’t think to simplify, just do the arithmetic:

(62 + 72) – (62 – 72) = (36 + 49) – (36 – 49) = 85 – (–13) = 85 + 13 = 98.

11. (7.5)    The perimeter of the quadrilateral in the figure is 30 (5 + 7 + 8 + 10). Then 4s = 30, where s is a side of the square, and s = 7.5.

12. (3.7) 123    To produce 40 gizmos takes 40 123 333 = 13,320 seconds. Since there are 60 seconds in a minute and 60 minutes in an hour, there are 60 123 60 = 3600 seconds in an hour; 13,320 ÷ 3600 = 3.7 hours.

**13,320 seconds ÷ 60 = 222 minutes, and 222 minutes ÷ 60 = 3.7 hours.

13. (145)    Since the average of ab, and 10 is 100, their sum is 300 (TACTIC E1). Then

a + b + 10 = 300 123 a + b = 290.

123

14. (1.21)    Use TACTIC 7. Since this is a percent problem, assume the rent last year was $100. Since 10% of 100 is 10, this year the rent went up $10 to $110. Now, 10% of 110 is 11, so next year the rent will go up $11 to $121. Finally, 121 is 1.21 123 100.

15. (36)

• Since the first digit must be odd, there are 3 choices for the first digit: 1, 3, 5.

• There are 2 choices for the last digit: one of the two odd integers not chosen as the first digit.

• There are 3 choices for the second digit: any of the three integers not yet used.

• There are 2 choices for the third digit: either one of the two integers not yet used.

• There is 1 choice for the fourth digit: the one integer still not used.

• So, by the counting principle, there are 3 × 2 × 3 × 2 × 1 = 36 integers that satisfy all three conditions.

16. (9.36) 123    The class average will be highest when all the grades are as high as possible. Assume that all 22 students who passed earned 10’s. Of the 3 who failed, 1 received a grade of 2; but assume that the other 2 students had 6’s, the highest failing grade. Then the total is 22 123 10 + 2 + 2 123 6 = 220 + 2 + 12 = 234, so the highest possible class average is 234 ÷ 25 = 9.36.

17. (6)    Let 2x and 7x represent the number of red and blue marbles, respectively, in jar I. Then in total there are 7x blue marbles and 14x red ones. Since there are 2x red marbles in jar I, there are 12x red marbles in jar II. Then there are 6 times as many red marbles in jar II as there are in jar I.

**Do the same analysis, except let x = 1. Then jar I contains 2 red and 7 blue marbles, whereas jar II contains 12 red ones.

18. (123)    Adding the fractions, we get 123 But it is given that ab is 3 times (a + b). Therefore, 123

Section 8 Critical Reading

  1. C.    To be the subject of a major exhibition would surely rescue a forgotten artist from obscurity (the state of being unknown).

(Cause and Effect Pattern)

  2. D.    If we see things in a distorted or altered fashion, our testimony is unreliable.

Note how the second clause serves to clarify or define the meaning of the missing word.

Remember: before you look at the choices, read the sentence and think of a word that makes sense. Likely Words: undependable, misleading.

(Definition)

  3. D.    People who shut themselves away from society are, by definition, hermits or recluses.

(Definition)

  4. B.    Heroic virtues include disregard or ignoring of death and fortitude or courage in the face of torture. Through it all, Bond remains nonchalant or cool.

(Examples)

  5. B.    If the code did not exist until 1846, it could not have been rescinded (canceled), presupposed (required as an already existing condition), or depreciated (disparaged) at that time.

It makes most sense that the code was promulgated or made known to the public by the AMA at that time.

(Definition)

  6. C.    The Romantic poets can be described as emotional; Arnold and the later “moralizing” Victorian era poets can be described as didactic (interested in teaching). Remember to watch for signal words that link one part of the sentence to another. The use of unlike in the opening clause sets up a contrast. The missing words must be antonyms or nearantonyms. You can immediately eliminate Choices A and B as synonyms or nearsynonym pairs.

(Contrast Signal)

  7. D.    The author mentions these terms as examples of what he means by the strange new language or idiosyncratic nomenclature of modern particle physics.

  8. D.    In his references to the elegance of the newly discovered subatomic structures and to the dance of Creation, the author conveys his admiration and wonder.

9. B.    “Matter’s heart,” where the physicist can observe the dance of Creation, is the subatomic world, the world of quarks and charms.

10. C.    In the opening sentence of the final paragraph, the author states that “the astronomers and the particle physicists are engaged in a vigorous dialogue.” Thus, he regards their talks as energetic or spirited. Choice A is incorrect. The author describes the talks as vigorous, not contentious (argumentative; quarrelsome). Choices B, D, and E are also incorrect; nothing in the paragraph suggests that the author regards the talks between the astronomers and particle physicists as unrealisticdistracting, or poetic.

11. D.    The image of the snake swallowing its tail suggests that the astronomers’ and physicists’ theories are, at bottom, one and the same. In other words, there is an underlying unity connecting them.

12. E.    The properties of the upsilon particle that implied it could not be made of up, down, strange, or charm quarks were its characteristics or attributes.

13. B.    Glashow is eager for the end of the hunt. His words (“last blessed one,” “the sooner...the better”) reflect his impatience.

14. E.    The keystone of the arch (the wedge-shaped block that is inserted last into the arch and locks the other pieces in place) completes the arch. By comparing the top quark to the keystone, the author of Passage 2 illustrates the importance of the top quark to subatomic theory.

15. D.    The physicists had to find the top quark because their theory depended on the top’s existence.

16. E.    The author of Passage 2 cites authorities (Glashow, Tollestrup) and uses similes (“like an arch”). She defines the Standard Model as the theoretical synthesis that reduced the zoo of subatomic particles to a manageable number. She poses a question about what makes certain particles more massive than others. However, she never denies a possibility.

17. C.    Physicists are familiar with the weight of a gold atom. In stating that the top was determined to weigh about as much as a gold atom, the author is illustrating just how hefty or massive a top quark is.

18. C.    The 1995 experiments succeeded: The physicists found the keystone to their arch. From this we can infer that the Standard Model was not disproved but instead received its validation.

19. B.    In lines 31–37, the author of Passage 1 develops a fanciful metaphor for the nature of matter. To him, subatomic matter is like a Bach fugue, filled with arpeggios. While the author of Passage 2 resorts to some figurative language (“Grail,” “keystone”) in attempting to describe the top quark, she is more factual than figurative: she never uses any metaphor as extended as the metaphor “the music of matter.” Thus, her most likely reaction to lines 31–37 would be to point out that this metaphor is too fanciful to be worthwhile.

Section 9 Mathematical Reasoning

  1. B.    Solve the given equation: 123

Add the fractions: 123

Multiply both sides by a: 3 = 12a

Divide both sides by 12: 123

**You can use TACTIC 5: backsolve; try choice C. If 123, so the left-hand side equals 9. That’s too small. Now, be careful: a fraction gets bigger when its denominator gets smaller (KEY FACT B4). Eliminate C, D, and E, and try a smaller value for a123 works.

  2. C.    If x = –5, then

2x2 – 3x – 7 = 2(–5)2– 3(–5) – 7 = 2(25) + 15 – 7 = 58.

  3. A.    The formula for the area of a circle is: A = 123r2

Divide both sides by 123123

Take the square root of each side: 123

The diameter is twice the radius: 123

**Let the radius of the circle be 1. Then the area is 123, and the diameter is 2. Which of the five choices is equal to 2 when A = 123? Only 123

  4. E.    Carefully read the values from the chart. Ann, Dan, Pam, Fran, and Sam read 1, 4, 2, 6, and 5 books, respectively. The sum is 18.

  5. D. 123    The average number of books read by the five members is the sum, 18 (calculated in the solution to question 3), divided by images. Three of the five members, or 60%, read more than 3.6 books.

  6. D. 123    If Laurie had to pay 30% of the value of her inheritance in taxes, she still owned 70% of her inheritance: 70% of 40% is 28% (0.70 123 0.40 = 0.28).

**Assume the estate was worth $100. Laurie received 40%, or $40. Her tax was 30% of $40, or $12. She still had $28, or 28%, of the $100 estate.

  7. E. 123    The months of the year form a repeating sequence with 12 terms in the set that repeats. By KEY FACT P2, the nth term is the same as the rth term, where r is the remainder when n is divided by 12.

555 ÷ 12 = 46.25 123 the quotient is 46. 46 123 12 = 552 and 555 – 552 = 3 123 the remainder is 3.

Therefore, 555 months from September will be the same month as 3 months from September, namely December.

  8. E.    The graph of y = –f (x) is the reflection in the x-axis of the graph of y = f (x).

123

Of the five choices, only (2, 0) is on this graph.

  9. B.    Write the given equation as: a3 = 3a

Since a is positive, divide both sides by aa2 = 3

Take the square root of each side: a = 123

**Use TACTIC 5: test the choices, starting with C.

10. A.    If e is the edge of the cube, the surface area, A, is 6e2 and the volume, V, is e3 (KEY FACTS M1 and M2). Then

123

123

11. A.    123

**If you use your calculator, you don’t need to change 123 Just enter

123. If you prefer, you can enter the exponents as .5 and .25.

12. D.    Let r = radius of circle I, and let R = radius of circle II. Then 2R is the diameter of circle II, and 2123 r is the circumference of circle I.

123

**Use TACTIC 6. Pick some easy-to-use number, such as 1, for the radius of circle I. Then the circumference of circle I is 2123, which is the diameter of circle II, and the radius of circle II is 123 (one-half its diameter). The area of a circle is given by A = 123 r2, so the area of circle I is 123(1) = 123, and the area of circle II is 123(1232) = 1233. Finally, the ratio of their areas is 123

13. C.    Exactly 3 of the numbers on the dart board are prime: 2, 3, and 31. Therefore, the probability that a dart lands on a prime is 123

(Remember: 1 is not a prime.)

14. C.    The elements of B that are in A are the perfect cubes between –50 and 50. There are 7 of them: –27, –8, –1, 0, 1, 8, 27.

15. A.    Use TACTIC 1: draw a diagram.

123

Form rectangle BCDE by drawing DE 123 AB . Then, BE = 9, AE = 16, and DE = 12. Finally, DA = 20, because right triangle AED is a 3-4-5 triangle in which the length of each side is multiplied by 4. If you don’t realize that, use the Pythagorean theorem to get DA:

(DA)2= (AE)2+ (DE)2= 162 + 122 = 256 + 144 = 400 so, DA = 20.

16. E.    If 3a = 4b = 5c, then 123 so

123

Then 123

**Use TACTIC 6: plug in easy-to-use numbers. The factors 3, 4, 5 suggest the number 60. Let a = 20, b = 15, c = 12. Then a + b = 35, so 35 = 12k and 123

Section 10 Writing Skills

  1. B.    Lack of parallelism. Choice B demonstrates proper parallel structure: vegetablesassortment, and loaf.

  2. C.    Lack of parallelism. The correlatives, not only … but also typically connect parallel structures. Choice C reflects the appropriate parallel construction.

  3. C.    Error in logical comparison. Compare renown with renown, not with a renowned painter.

  4. D.    Error in subject-verb agreement. The subject, diseases, is plural. The verb should be plural as well. Change has become to have become.

  5. E.    Error in subject-verb agreement. Do not be misled because the subject follows the verb. Here, the subject, list, is singular; the verb should be singular as well. Change There are to There is.

  6. C.    Run-on sentence. Do not link two independent clauses with a comma. The addition of the connective and in Choice C corrects the error.

  7. A.    Sentence is correct.

  8. D.    Error in subject-verb agreement. The subject, spread, is singular; the verb should be singular as well. Change are evolving to is evolving.

  9. D.    Ambiguous reference. The children were removing the carrots’ skins, not their own skins.

10. C.    Lack of parallelism. Choice C supplies the appropriate parallel structure.

11. A.    Sentence is correct.

12. D.    Wordiness. The suggested revision tightens this ineffective compound sentence in two ways: first, it eliminates the connective and; second, it repeats the phrase a scent to emphasize its importance.

13. E.    Sentence fragment. The introduction of a subject (She) and the substitution of a main verb (attempted) for the participle Attempting result in a complete sentence.

14. A.    Sentence is correct.